exam 1 nurs 102 spring

Réussis tes devoirs et examens dès maintenant avec Quizwiz!

right lobe of liver, gallbladder, portions of the pancreas, small intestines, and colon

what organs are located in the right upper quadrant (RUQ)

cerebellum

what part of the brain is responsible for coordinating of motor activities of movement

cerebral cortex

what part of the brain is responsible for initiating voluntary motor activity

mitral valve and tricuspid valve

what valves close during s1

tachypnea

when the patient has above 20 breaths per minute

fingertips

which part of the hand is best from being able to feel the texture of a patient's skin

bell

which part of the stethoscope is used to hear low pitched sounds such as murmurs

aortic valve and pulmonary valve

which valves close during s2

contractures

this is the progressive shortening of a muscle and loss of joint mobility resulting from fibrotic changes in tissues and surrounding joints

sitz bath

this is used to cleanse, soothe and reduce inflammation or perineal or vaginal area after child birth or local irritation or hemorrhoids and fissures

varicosities

this is veins that seem swollen or engorged

4+

this is when a patients pulse is bounding

1+

this is when a patients pulse is considered to be thready could be a sign of dehydration

pulse deficit

this is when a ventricular contraction does not perfuse the body adequately

arterial insufficiency

this is when oxygenated blood is not getting to the lower extremities (dusty pale smooth to touch skin will look shinny and thin)

biot's-ataxic breathing

this is when the patient has an abnormal patter of breathing characterized by complete irregularity of breathing with irregular pauses and increasing periods of apnea

dyspnea

this is when the patient has difficult or laboring breaths

eupnea

this is when the patient has normal good unlaboring breaths

apnea

this is when the patients breathing repeatedly stops and starts

tachycardia

this is when the patients pulse is above 100

bradycardia

this is when the patients pulse is below 60

Gooseflesh

this looks like a pimply state of the skin with the hairs erect, produced by cold or fright

Vehicle transmission

this mode of transmission involves the transfer of microorganisms by way of vehicles or contaminated items that transmit pathogens

Airborne transmission

this mode of transmission occurs when fine particles are suspended in the air for a long time or when dust particles contain pathogens.

Droplet transmission

this mode of transmission occurs when mucous membranes of the nose, mouth, or conjunctiva are exposed to secretions of an infected person who is coughing, sneezing, or talking infections could reach 6 feet or more from the source

Acute

this phase of illness occurs when specific symptoms appear.

Mode of transmission

this refers to the way in which the organism moves or is carried from the source's portal of exit

romberg test

this test assess the patients motor functions (mainly balance)

canes

this type of ambulation assistive devices are useful for patients who can bear weight but need support for balance or who have decrease strength in one leg

Isotonic exercise

this type of exercise is a dynamic form of exercise with constant muscle tension, muscle contraction, and active movement

anaerobic exercise

this type of exercise is a physical exercise intense enough to cause lactate to form. during these high-intense bouts of exercises, the body demands more oxygen and must rely on energy breakdown from the muscle stores.

aerobic exercise

this type of exercise requires oxygen to use the energy provided by the metabolic activities of the skeletal muscles

isometric exercise

this type of exercise, is a static exercise by which the client tenses a muscle, holding it stationary while maintaining the tension

hemiplegic gait

this type of gait happens when one leg is paralyzed or neurologically damaged so the leg is dragged or swung around to propel it forward

festinating gait

this type of gait is characterized by walking on the toes as being pushed. also characterized by a quickening and shortening of normal strides. This phenomenon is commonly observed in patients with Parkinson's

external respiration

this type of respiration deals with taking O2 into the body and elimination CO2 from the body

internal respiration

this type of respiration deals with the use of oxygen, the production of carbon dioxide and the exchange of O2 and CO2 between the cells and the blood

isometric exercise

this type of static exercise by which the patient tenses a muscle holding it stationary while maintaining the tension (done in static positions)

vehicle transmission

this type of transmission involves the transfer of microorganisms by way of vehicles or contaminated items that transmit pathogens

airborne transmission

this type of transmission is when fine particles are suspended in the air for a long time or when dust particles contain pathogens

droplet transmission

this type of transmission is when the mucous membrane of the nose, mouth or conductive are exposed to recreations of an infected person who is coughing sneezing or talking

contains portions of small intestines and colon, left ovary and fallopian tube, and left ureter

what organs are located in the left lower quadrant (LLQ)

left lobe of liver, spleen, stomach, portions of the pancreas, small intestine and colon

what organs are located in the left upper quadrant (LUQ)

portions of small intestines and colon, right ovary and fallopian tube, appendix, and right ureter

what organs are located in the right lower quadrant (RLQ)

basal ganglia

what part of the brain is responsible for maintaining posture

what causes the symptom? what makes it worse?

what question does the P stand for in the PQRST

how does the symptom feel, look or sound?

what question does the Q stand for in the PQRST

where is the symptom located?

what question does the R stand for in the PQRST

how does the symptom rate on a severity scale

what question does the S stand for in the PQRST

when did the symptom begin

what question does the T stand for in the PQRST

airborne infection

what type of infection is tuberculosis (TB)

radial

when it comes to a patients apical and radial pulse which pulse would be lower?

prehypertension

when taking a patients blood pressure a reading with at systole of 121-139 and a diastole of 81-99 would be considered to be in

hypertension

when taking a patients blood pressure a reading with at systole of 140 and higher and a diastole of 100 and higher the patient would be considered to be in

false low

when taking a patients blood pressure and the cuff is to large what kind of reading will you get

false high

when taking a patients blood pressure and the cuff is to small what kind of reading will you get

axillary

when taking a patients temperature this is said to be the least accurate

tympanic

when taking a patients temperature this is said to be the most accurate

rectal

when taking a patients temperature this is said to be the patients core temp

bradypnea

when the patient has below 12 breaths per minute

5th intercostal space midclavicular line

where is the PMI located (Point of maximal impulse)

dorsum

which part of the hand is best for being able to feel the vibrations transmitted through the body (fremitus)

chorea

this is spontaneous brief involuntary muscle twitching of the limbs or facial muscles

contact transmission

this is the most frequent type of transmission and is by direct or indirect contact

edema

this is the presence of fluid in the intercostal spaces that can be caused by fluid in subcutaneous tissues

how does the symptom rate on a severity scale of 1 to 10, with 10 being the most extreme

what does the S stand for in the PQRST

when did the symptoms begin? how often does it occur? sudden or gradual?

what does the T stand for in the PQRST

subjective, objective, assessment, plan, intervention, evaluation, revision

what does the nemonic SOAPIER stand for

breezy clear breath sound, decreased respiratory rate

what effects do you see on the respiratory system when you exercise

systole of 90-120 and a diastole of 60-80

what is a normal blood pressure reading

95-100

what is a normal reading for a patient when taking their oxygen saturation

to increased concentration of carbon dioxide

what is the is the strongest stimulus to breathe more deeply and more frequently?

4

what level of self-care if the patent completely depends on another to preform self-care activities

3

what level of self-care if the patient requires assistance or supervision from another and uses devices or equipment

spasticity

this is increased muscle tone

micturition

Urination

2

what level of self-care if the patient requires assistance or supervision from another to complete self-care activities

1

what level of self-care if the patient uses equipment or devices to perform self-care activities independently

crepitus

crunching or grating sound because of insufficient joint lubrication

0

what level of self-care is the patient at if they are independent in all self-care activities

sepsis

poisoning of body tissues, systemic inflammatory state triggered by two things: widespread immune response and microbial distraction of tissue

Adds depth to existing information

A nurse is assessing a client with chronic back pain and asking specific questions to obtain a focus assessment. Which of the following are features of a focus assessment? -Adds depth to existing information -Suggests possible problems -Provides breadth for future comparisons -Gives a comprehensive volume of data

Neurologic system, Musculoskeletal system

A nurse is assessing a patient for safety upon discharge following a severe cerebrovascular. Which of the following systems would be a priority for this patient's safety? Select all that apply: -Neurologic system -Cardiovascular system -Skin integrity -Musculoskeletal system

slightly below the left nipple

A nurse is assessing the apical heart rate of a healthy person. In order to hear the heartbeats loud and clear, where should the nurse place the stethoscope? -over the sternum at the midpoint between the nipples -on the center of the rib cage -slightly below the left nipple -4 in (10 cm) below the left clavicle

The reading is erroneously high

A nurse is assessing the blood pressure on an obese woman. What error might occur if the cuff used is too narrow? -The reading is erroneously high. -It will be difficult to pump up the bladder. -The pressure on the cuff will be painful. -The reading is erroneously low.

Auscultation

A nurse is assessing the bowel sounds of a client who has Crohn's disease. What assessment technique would the nurse use? -Auscultation -Inspection -Palpation -Percussion

5.5 L/min

A nurse is assessing the cardiac output of a client at the health care facility. What would the nurse identify as the average cardiac output in a resting person? -5.5 L/min -8.5 L/min -7.5 L/min -6.5 L/min

Auscultate breath sounds

The nurse is caring for a patient who is coughing up blood. What is the nurses Next action? -Apply 2 liters nasal cannula of oxygen -Collect a sputum culture -Auscultate breath sounds -Notify Health Care Provider

droplet

The nurse is caring for a pediatric client with whooping cough. Which precautions will the nurse begin? -none -contact -airborne -droplet

flushed

The skin of a client with fever and hypertension appears pink and is documented as -Pallor -Ecchymosis -Flushed -Erythema

Standard precautions

Which of the following terms best describes the synthesis of major features of decreasing risk of transmission of bloodborne pathogens and body substance isolation? -Universal precautions -Medical asepsis -Standard precautions -Protective isolation

Carotid

Which peripheral pulse site is generally used in emergency situations? -Carotid -Radial -Temporal -Apical

Pathogenic

A 12-year-old is being hospitalized for pneumonia. The nurse receives the client's culture and sensitivity report on her tracheal aspirate. The client is infected with a strain of Streptococcus pneumoniae, which is particularly prone to cause infections, also referred to as what? -Pathogenic -Specific -Source -Virulent

decrease appetite

A 45-year-old man is interested in starting an exercise program. The nurse informs him that exercise does not: -enhance mood. -improve sleep quality. -decrease appetite. -prevent constipation.

Neuromuscular

A 45-year-old woman has multiple sclerosis. She is able to perform most functions of self-care but recently she has been having problems with balance, which has made it hard to get dressed. Which factor is affecting this client's ability to perform self-care? -Neuromuscular -Motivation -Cognitive -Sensory

Consult nursing and medical literature

A 50-year-old female client is admitted to a hospital unit with the diagnosis of scleroderma. The nurse is unfamiliar with this condition. What is the nurse's best source of information? -Consult nursing and medical literature. -Consult with the client. -Read the client's chart. -Consult with the client's doctor.

Consult nursing and medical literature

A 50-year-old female client is admitted to a hospital unit with the diagnosis of scleroderma. The nurse is unfamiliar with this condition. What is the nurse's best source of information? -Read the client's chart. -Consult nursing and medical literature. -Consult with the client. -Consult with the client's doctor.

medications listed on the client's medication administration record (MAR)

A 55-year-old client has just undergone surgery for a knee replacement. He asks the nurse if he can shave because his face is itching from the stubble. What information is a priority for the nurse to verify prior to shaving the client? -the last time shaving was performed because clients can only shave twice weekly in the hospital -client's allergies to soap since shaving cream is contraindicated in the hospital -cultural views and attitudes toward facial hair and grooming -medications listed on the client's medication administration record (MAR)

"Be sure to drink 8 ounces of water when you take alendronate, and take it on an empty stomach."

A 59-year old female client reports to the nurse that she recently began taking alendronate and has been having stomach cramping, nausea, and diarrhea. How will the nurse educate the client? -"Discontinue taking the medication immediately." -"Take this medication monthly instead of weekly." -"Be sure to drink 8 ounces of water when you take alendronate, and take it on an empty stomach." -"These side effects are normal, so do not worry about them."

ensuring that each necessary element of catheter care is consistently provided

A care facility has introduced care bundles for clients who have indwelling urinary catheters. When describing the use of care bundles, what benefit should be explained? -ensuring that each necessary element of catheter care is consistently provided -aligning practices with the Patient Protection and Affordable Care Act (ACA) -designating one nurse who is ultimately accountable for catheter care in the facility. -prioritizing the client's preferences in the catheter care that is provided

Dilute with water or milk

A caregiver of a toddler has called the poison control nurse to report that the child licked a small amount of petroleum jelly. The caregiver states that the toddler is sitting on the floor, watching a cartoon, and playing with a toy. Which information will the poison control nurse provide? -Administer laxative. -Induce vomiting. -Call 9-1-1. -Dilute with water or milk.

to determine the location, size, and density of underlying structures

A nurse conducting physical assessment for a client is using the percussion technique. What is the purpose of using this technique? -to determine the location, size, and density of underlying structures -to check the skin temperature and moisture -to assess the sounds from the heart, lungs, and abdomen -to assess the mobility of normal tissues and unusual masses

"Hypertension can be caused by many things, including anxiety and obesity."

A client asks the nurse if there are risk factors for hypertension. What is the most appropriate nursing response? -"Hypertension can be caused by many things, including anxiety and obesity." -"Hypertension happens when medication does not control the symptoms." -"We don't know what causes hypertension, so we can't identify risk factors." -"Heart failure is the only cause for hypertension."

Focused assessment

A client comes to her health care provider's office because she is having abdominal pain. She has been seen for this problem before. Which type of assessment would the nurse perform? -Emergency assessment -Focused assessment -Initial assessment -Time-lapse assessment

Admission assessment

A client comes to the acute care facility for diagnostic testing and elective surgery. Which type of assessment would the nurse most likely complete? -Emergency assessment -Time-lapse reassessment -Admission assessment -Focus assessment

Focused

A client comes to the emergency department with a productive cough and an elevated temperature. Which type of assessment would the nurse most likely perform on this client? -Focused -Emergency -Head-to-toe -Time-lapse

Intact skin and mucous membranes protect against microbial invasion

A client comes to the emergency department with major burns over 40% of his body. Although all of the following are true, which one would provide the rationale for a nursing diagnosis of Risk for Infection? -Age, race, sex, and hereditary factors influence susceptibility to infection. -White blood cells provide resistance to certain pathogens. -Intact skin and mucous membranes protect against microbial invasion. -Stress may adversely affect normal defense mechanisms.

Colonization

A client has a concentration of Staphylococcus aureus located on his skin. He is not showing signs of increased temperature, redness, or pain at the site. The nurse is aware that this is a sign of a microorganism at which stage? -Colonization -Infection -Disease -Bacteremia

The client will state how to safely take the prescribed antibiotic

A client has a nursing diagnosis of Deficient Knowledge related to prescribed antibiotic therapy. Which outcome would the nurse identify as most appropriate? -The client demonstrates the proper technique for hand hygiene. -The client will state how to safely take the prescribed antibiotic. -The client will identify signs and symptoms of worsening infection. -The client will verbalize measures appropriate to minimize infection transmission.

administering a unit of packed red blood cells

A client has been admitted with a gastrointestinal bleed. Two nurses should perform checks before which intervention is performed? -drawing a blood sample for analysis of hemoglobin levels -placing the client on "n.p.o." (nothing by mouth) status -inserting a peripheral intravenous catheter in the client's forearm -administering a unit of packed red blood cells

cross-country skiing

A client has been prescribed exercise at the metabolic energy equivalent (MET) of 8. Which exercise type will the nurse recommend? -cross-country skiing -mowing the lawn -lifting 50 pounds -raking leaves

Activity Intolerance

A client has chronic obstructive pulmonary disease (COPD) and is unable to perform basic self-care activities or activities of daily living (ADLs). What would be an appropriate nursing diagnosis? -Risk for Injury: Pathologic Fractures -Altered Thought Processes -Altered Tissue Perfusion -Activity Intolerance

Ensure clear, accurate communication between each of the team members.

A client has gone into cardiac and respiratory arrest and resuscitation is being attempted at the client's hospital bedside. The nurse recognizes the high potential for errors in this emergency setting. Which action is most likely to prevent adverse incidents in this situation? -Ensure clear, accurate communication between each of the team members. -Begin documentation of the care provided once the outcome of the resuscitation is known. -Ensure that the client's family members are escorted from the bedside. -Double check every medication that is administered to the client.

wound infection

A client has had abdominal surgery. During a follow-up visit to the surgeon on the tenth postoperative day, the client exhibits a fever. The nurse suspects which of the following as the most likely cause? -Stress of surgery -Pneumonia -Urinary tract infection -Wound infection

completing a mental status assessment

A client is brought to the emergency department after being hit in the head with a bat during a softball game. What is the nurse's priority assessment? -completing a mental status assessment -checking peripheral pulses -auscultating the heart and lungs -assessing vital signs

Client's wife

A client is brought to the emergency department in an unconscious condition. The client's wife hands over the previous medical files and points out that the client had suddenly fallen unconscious after trying to get out of bed. Which of the following is a primary source of information? -Client's wife -Test results -Assessment data -Medical documents

"Two separate negative test results will clear you of the infection."

A client is diagnosed with methicillin-resistant Staphylococcus aureus (MRSA) infection in the respiratory system. He wants to know if he will eventually be cleared of MRSA. Which response is appropriate? -"Once diagnosed with MRSA, you will always have the infection in you." -"Two separate negative test results will clear you of the infection." -"You will always have a positive result after an infection." -"Your sputum will be tested after the antibiotic administration to determine your infection level."

Tepid water

A client is in the fever phase. His temperature remains significantly elevated. The nurse is preparing to implement sponge bathing. Which type of water would the nurse most likely use? -Tepid water -Warm water -Cool water -Ice water

"Try to avoid putting too much pressure on your armpits with the tops of the crutches."

A client is preparing to mobilize for the first time following the surgical removal of a bunion on her left foot. How should the nurse instruct the client to ambulate with her crutches? -"Keep your elbows well away from your sides in order to keep yourself as stable as possible." -"Keep your crutches as close as possible to your feet when you're walking." -"When you rise from a chair, use your left foot to stabilize yourself." -"Try to avoid putting too much pressure on your armpits with the tops of the crutches."

Apical

A client is taking medications to treat a heart dysrhythmia. Which site should be used to assess the pulse in this client? -Radial -Apical -Dorsalis pedis -Brachial

Surgical asepsis technique

A client is to have an indwelling urinary catheter inserted. Which precaution is followed during this procedure? -Medical asepsis technique -Strict reverse isolation -Surgical asepsis technique -Droplet precautions

oral fluids

A client presents to the Emergency Department with a temperature of 100.6F (38.1°C) and BP of 108/60 mm Hg. What intervention does the nurse anticipate providing? -ibuprofen -cooling blanket -oral fluids -acetaminophen

Client himself

A client reports to a health care facility with reporting abdominal pain and vomiting. The client's wife informs the nurse that the client had gone out for dinner the previous night. Which of the following would be the primary source of assessment data? -Client himself -Client's wife -Test reports -Client's friends

public health department

A client was admitted to the emergency department with a confirmed diagnosis of tuberculosis. To whom should the nurse report this diagnosis? -client's employer -public health department -client's family -Health Canada

Hold sterile objects above waist level to prevent inadvertent contamination

A nurse follows surgical asepsis techniques for inserting an indwelling urinary catheter in a client. What is an accurate guideline for using this technique? -Consider the outside of the sterile package to be sterile. -Consider the outer 3-in (8-cm) edge of a sterile field to be contaminated. -Hold sterile objects above waist level to prevent inadvertent contamination. -Open sterile packages so that the first edge of the wrapper is directed toward the nurse.

"Stress leads to increased secretion of cortisol, which suppreses your immune response."

A client who comes to the clinic asks the nurse, "Somebody told me that stress increases my risk for infection. How does this happen?" Which response by the nurse would be most appropriate? -"Stress causes the body's normal immune response to turn on itself." -"Stress leads to increased secretion of cortisol, which suppreses your immune response." -"Stress leads to a deterioration in the skin's barrier line of defense." -"Stress causes body fluids to accumulate, which leads to bacterial growth."

"The common cold is a virus and will not respond to antibiotics."

A client who has been diagnosed with a cold is upset that antibiotic therapy was not prescribed. Which nursing response is most appropriate? -"Sometimes antibiotics work for colds and sometimes they do not." -"The common cold is a virus and will not respond to antibiotics." -"We can ask the PCP for an antiviral medication." -"Antibiotics have too many side effects anyway."

Bacteria

A client who has had abdominal surgery develops an infection in the wound while still hospitalized. Which agent is most likely the cause of the infection? -Virus -Spores -Bacteria -Fungi

contact isolation

A client with Clostiridium difficile requires what type of isolation

"Your white blood cells have increased in the area."

A client with a wound infection asks the nurse, "What causes this puslike drainage in my wound?" Which response by the nurse would be most appropriate? -"It results from the swelling caused by the pain of the inflammation. -"Metabolism in your wound tissues is increased." -"Your white blood cells have increased in the area." -"It's just a sign that your wound is infected."

WBC of 25,000 mcL

A client with cancer has been receiving chemotherapy for the past few weeks. The nurse is concerned about infection and is reviewing the white blood cell count (WBC) in the chart. Which result supports this concern? -WBC of 25,000 mcL -WBC of 10,500 mcL -WBC of 5,500 mcL -WBC of 7,500 mcL

both crutches are moved forward, one or both legs are advanced beyond crutches

A client with paralysis has been prescribed swing-through crutch-walking. Which teaching regarding gait pattern will the nurse provide? -one crutch, opposite foot, other crutch, remaining foot -both crutches are moved forward, one or both legs are advanced beyond crutches -both crutches move forward, followed by the weight-bearing leg -one crutch and opposite foot moved in unison, followed by remaining pair

"I hold the boxes away from my body so I don't drop them on my feet."

A client works in a warehouse and has been having low back pain. Which statement would indicate the need for more education regarding safe lifting? -"I try to rest between periods of lifting." -"I hold the boxes away from my body so I don't drop them on my feet." -"I stand with my feet apart so I have a better stance when I lift." -"I bend with my knees when I pick up boxes."

10,000

A community health nurse is teaching a 22-year-old healthy client about activity monitoring with a self-monitoring motion sensing device. The nurse will recommend the client set a daily goal for how many steps? -5000 -1000 -10,000 -15,000

6000

A community health nurse is teaching a 42-year-old sedentary client about activity monitoring with a self-monitoring motion sensing device. The nurse will recommend the client set an initial daily goal for how many steps? -6000 -7600 -14,000 -11,000

T-lymphocytes

A group of nursing students is reviewing the various white blood cells and how they function in infection. The students demonstrate understanding of the information when they identify which cell as important in synthesizing immunoglobulins? -Eosinophils -Monocytes -T-lymphocytes -Neutrophils

having every employee focus on safety, not only direct care providers.

A health care institution's most recent strategic plan includes a commitment to creating a culture of safety. The organization can best meet this commitment by: -publicizing events that threatened client safety, while still maintaining confidentiality. -ensuring that the care clients receive is based on the latest research evidence. -rotating staff members through various sites in order to reduce complacency. -having every employee focus on safety, not only direct care providers.

The rate of postoperative complications is significantly higher than national averages.

A hospital is being evaluated by the Centers for Medicare & Medicaid Services. Which of these findings from the evaluation may result in a reduction in the hospital's reimbursement under the value-based purchasing (VBP) program? -The rate of postoperative complications is significantly higher than national averages. -The hospital has a policy of using generic drugs rather than brand-name drugs whenever possible. -The hospital is not using the latest version of its electronic health records software. -The ratio of registered nurses to licensed practical nurses is significantly lower than in other similar-sized hospitals.

enhancing the communication that takes placed between team members

A hospital unit's safety committee is evaluating current practices while performing a root cause analysis of an error. Which initiative on the unit has the greatest potential to reduce the number of errors? -increasing the mean education level of staff on the unit -implementing more serious consequences for staff members who commit errors -making a public commitment to clients that no errors will take place on the unit -enhancing the communication that takes placed between team members

Use two unique client identifiers before giving medications.

A hospital's quality improvement committee is adapting the hospital's policies and procedures to align with the Joint Commission 2015 Hospital National Patient Safety Goals. Which of the following is an explicit focus of the 2015 goals? -Screen every new client for intimate partner violence. -Eliminate the use of shared client rooms to prevent healthcare-acquired infections (HAIs) -Use two unique client identifiers before giving medications. -Identify clients at high risk of developing postoperative complications.

The laboratory assistant can only retrieve patient records but cannot view the details

A laboratory assistant who is trying to view the electronic record of a client's personal history gets an error message, "You are not authorized to view this information." What is the reason for this message? -The laboratory assistant does not have the correct password. -The laboratory assistant can only retrieve patient records but cannot view the details. -The laboratory assistant does not have the correct access number. -The laboratory assistant is trying to view archived data.

11 p.m.

A nurse has applied restraints to a client as ordered at 9 p.m. The nurse adheres to the guidelines for restraint use by removing them at which time? -10:30 p.m. -11 p.m. -9:30 p.m. -10 p.m.

The nurse made a similar insulin error 4 months ago and has also made an oral medication error

A nurse has approached the unit manager and admitted to giving a client an incorrect dose of insulin. Which aspect of this event would suggest that it constitutes at-risk behavior rather than a human error? -The unit was exceptionally busy during the shift when the nurse made the error. -The nurse made a similar insulin error 4 months ago and has also made an oral medication error. -The nurse manager suspects that the nurse is not telling the truth about the timing of the error. -The nurse is a recent graduate and began working on the unit only 5 months ago.

Wipe with isopropyl alcohol

A nurse has been asked to record a client's body temperature every hour using a digital thermometer. After recording the temperature, the nurse has to clean the thermometer. Which measure should the nurse follow to clean the thermometer? -Soak in isopropyl alcohol. -Soak in water mixed with alcohol. -Wash with soap and water followed by alcohol. -Wipe with isopropyl alcohol.

Confirm in the client's health record that a change in dose has been ordered and then administer the new dose

A nurse has been caring for a client with a diagnosis of heart failure for several weeks, and the client's prescribed dose of digoxin has been unchanged since admission. However, after scanning the bar code on the medication prior to this morning's dose, the nurse notes that the computer is prompting a higher dose than on previous days. What is the nurse's most appropriate response? -Administer the usual dose and then document the discrepancy in the client's health record. -Administer the dose that is specified by the system and then consult with the primary care provider. -Confirm in the client's health record that a change in dose has been ordered and then administer the new dose. -Administer the usual dose and then speak with the client's primary care provider about the discrepancy.

the hospital's health and safety committee

A nurse has completed an incident report after a client tripped on an electrical cord in the hospital hallway and had a fall. The incident report will most likely be analyzed by: -the state's board of nursing. -the client's primary care provider. -the Occupational Safety and Health Administration (OSHA). -the hospital's health and safety committee.

Remove gloves, wash hands, face shield, gown, mask, and wash hands

A nurse has completed caring for a patient with MRSA in the sputum. As she exits the room, what is the most appropriate way to remove her PPE and wash hands? -Remove gloves, wash hands, remove gown, face shield, and mask -Remove gloves, wash hands, face shield, gown, mask, and wash hands -Remove gloves, gown, face shield, mask, and wash hands -Remove gown, wash hands, gloves, mask, goggles

Assess the client's health status

A nurse has entered a client's room and the client is lying on the floor next to her bed. What is the nurse's priority action? -Inform the primary care provider. -Assess the client's health status. -Complete an incident report. -Document the adverse event.

2,500 cells/mm3

A nurse in an oncology care unit is reviewing the laboratory test results of several clients scheduled to receive chemotherapy. The nurse determines that the client with which leukocyte count will most likely have the chemotherapy withheld? -2,500 cells/mm3 -5,800 cells/mm3 -7,500 cells/mm3 -9,800 cells/mm3

18,000 cells/mm

A nurse in an oncology care unit is reviewing the laboratory test results of several clients. The nurse identifies that the client with which leukocyte count most likely has an infection? -18,000 cells/mm -10,000 cells/mm -5,000 cells/mm -8,000 cells/mm

-Skin -Mouth -Vagina

A nurse is administering medication to an older adult client with candidiasis. Which sites are most common for candidiasis? Select all that apply. -Stomach -Skin -Mouth -Vagina -Bone

Orient the client to the room and environment thoroughly upon admission

A nurse is admitting a client to a geriatric medicine unit following the client's recent diagnosis of acute renal failure. Which nursing action is most likely to reduce the client's chance of experiencing a fall while on the unit? -Administer pain medications sparingly in order to minimize cognitive or musculoskeletal side effects. -Provide the client with a bedpan to reduce the need to transfer to a commode or washroom. -Orient the client to the room and environment thoroughly upon admission. -Place the client in a shared room with a client who is stable and oriented.

Falls from stairs, Accidental drowning, Electrocution from outlets

A nurse is applying prescribed medicine on the bruises of a toddler who injured himself after falling off a table. The nurse knows that infants and toddlers are vulnerable to injuries related to which type of injuries? Select all that apply. -Ingestion of toxic medicine -Accidental drowning -Electrocution from outlets -Play-related injuries -Falls from stairs

interview

A nurse is assessing a client admitted to the health care facility with angina. Which method would be most appropriate for the nurse to use to collect subjective data? -Scale -Stethoscope -Interview -Laboratory studies

Client is unable to communicate basic needs and cannot perform hygiene measures with left hand

A nurse is assessing a client admitted to the hospital with reporting left-sided weakness and difficulty speaking. Which assessment contains the data that best represent a nursing assessment? -Client is unable to communicate basic needs and cannot perform hygiene measures with left hand. -Brain scan shows evidence of a clot in the middle cerebral artery. -Neurologic examination reveals partial paralysis and aphasic speech. -Left-sided weakness and speech deficit indicate probable stroke.

Palpate the liver for enlargement.

A nurse is assessing a client and observes jaundice on the skin and hard palate on the sclera bilaterally. What is the appropriate action of the nurse? -Auscultate the lungs for crackles. -Percuss the spleen for tenderness. -Palpate the liver for enlargement. -Assess the client's temperature.

Palpate the liver for enlargement

A nurse is assessing a client and observes jaundice on the skin and hard palate on the sclera bilaterally. What is the appropriate action of the nurse? -Percuss the spleen for tenderness. -Auscultate the lungs for crackles. -Palpate the liver for enlargement. -Assess the client's temperature.

gown and gloves

Which personal protective equipment (PPE) should the nurse don to enter the room of a client who is diagnosed with Clostridium difficile? -goggles and gloves - respirator mask and gown -gown and gloves -mask and shoe covers

Document normal breath sounds

A nurse is assessing the lungs of a client and auscultates soft, low-pitched sounds over the base of the lungs during inspiration. What would be the nurse's next action? -Recommend testing for pneumonia. -Assess for asthma. -Document normal breath sounds. -Suspect an inflamed pleura rubbing against the chest wall.

Pulse is felt with difficulty and disappears with slight pressure

A nurse is assessing the pulse volume of a client with influenza. The nurse notes that the client has a thready pulse. Which of the following is a description of a thready pulse? -Pulse is felt easily, and moderate pressure causes it to disappear. -Pulse is strong and remains strong despite moderate pressure. -Pulse is strong, and light pressure causes it to disappear. -Pulse is felt with difficulty and disappears with slight pressure.

Auscultate the lung sounds and count respirations

A nurse is assessing the respirations of a 60-year-old female client and finds that the client's breaths are so shallow that the respirations cannot be counted. What would be the appropriate initial nursing intervention in this situation? -Administer oxygen. -Notify the primary care provider. -Perform a pain assessment. -Auscultate the lung sounds and count respirations.

Erythema

A nurse is assessing the skin of a client who had been on a hiking trip and developed a number of inflamed red patches on his hands and face as an allergic reaction. How should the nurse document this finding? -Pallor -Ecchymosis -Flushed -Erythema

The shoulder and upper back curves forward.

A nurse is assessing the spine of a client with kyphosis. What would the nurse expect to observe about the client's posture? -A portion of the spine is curved to the side laterally. -The shoulder and upper back curves forward. -The lumbar region tends to curve inward. -The sacral region tends to turn outward.

The shoulder and upper back curves forward

A nurse is assessing the spine of a client with kyphosis. What would the nurse expect to observe about the client's posture? -The shoulder and upper back curves forward. -The sacral region tends to turn outward. -The lumbar region tends to curve inward. -A portion of the spine is curved to the side laterally.

The nurse uses assistive devices when lifting more than 35 lb (16 kg) of client weight

A nurse is assisting client from a bed to a wheelchair. Which nursing action is appropriate? -The nurse grabs and holds the client by his arms. -The nurse administers pain medication following the transfer. -The nurse discourages the client from helping with the transfer. -The nurse uses assistive devices when lifting more than 35 lb (16 kg) of client weight.

to identify a life-threatening problem

A nurse is assisting with lunch at a nursing home. Suddenly, one of the residents begins to choke and is unable to breathe. The nurse assesses the resident's ability to breathe and then begins CPR. Why did the nurse assess respiratory status? -to practice respiratory assessment skills -to establish a database for medical care -to facilitate the resident's ability to breathe -to identify a life-threatening problem

Crackles are audible in the posterior bases bilaterally and they are abnormal.

A nurse is auscultating the lungs of a client during a physical exam. The nurse notes low-pitched, soft breath sounds over the posterior middle lobes with intermittent, high-pitched, popping sounds in the posterior lower lobes, primarily during inspiration. What is the nurse's correct interpretation of these findings? -Pleural friction rub is occurring in the posterior middle lower lobes. -Bronchovesicular breath sounds are audible in the posterior lobes. -Gurgling is occurring in the lower posterior lobes indicating the client needs to cough. -Crackles are audible in the posterior bases bilaterally and they are abnormal.

5,850 mL (5,850 × 109/L)

A nurse is calculating the cardiac output of an adult with a stroke volume of 75 mL (75 × 109/L) and a pulse of 78 beats/min. What number would the nurse document for this assessment? -5,550 mL (5,500 × 109/L) -5,000 mL (5,000 × 109/L) -6,000 mL (6,000 × 109/L) -5,850 mL (5,850 × 109/L)

Proteins

A nurse is caring for a client who has a lack of appetite. What is most likely to influence a client's core body temperature? -Vitamins -Fiber -Proteins -Minerals

obtaining rectal temperatures

A nurse is caring for a client who has neutropenia resulting from chemotherapy. Which precaution would be least appropriate to include when caring for this client? -avoiding razors with blades -encourage wearing a mask when out of the room -providing gentle oral care -obtaining rectal temperatures

monitoring the client's health status frequently and thoroughly

A nurse is caring for a client who is being treated for complications of diabetes. Which action by the nurse best reduces the client's risk of experiencing an adverse outcome while receiving care? -ensuring that the care provided is efficient and timely -documenting the care that the client receives in a timely and detailed manner -monitoring the client's health status frequently and thoroughly -delegating care appropriately and working closely with unlicensed care providers

Ask housekeeping personnel to clean the client's room last.

A nurse is caring for a client with meningococcal meningitis in a private room located close to the nursing unit of the health care facility. Which infection control measure should the nurse take? -Ask housekeeping personnel to clean the client's room last. -Flush the solutions used for cleaning the room down the sink. -Keep the doors and windows open for cross-ventilation. -Allow only one visitor at any given time to meet the client.

1500

A nurse is caring for a client, age 4 years, who is being treated for osteomyelitis in his left femur. He is on a 28-day course of IV vancomycin to be administered daily at 1300. Today is day 3 of treatment, and the pharmacist asks the nurse to draw a peak vancomycin level. What would be the most appropriate time to draw this blood? -Wait until day 5 of treatment. -2000 -1200 -1500

Assess the patients pain level and pain characteristics

A nurse is caring for a patient who is exhibiting signs and symptoms of a myocardial infarction (MI). What should the nurse do FIRST? -Assess anxiety level and provide emotional support -Prepare patient for a cardiac catheterization -Notify the Health Care Provider -Assess the patients pain level and pain characteristics

Perform a focused neuro assessment

A nurse is caring for a patient who presents with slurred speech. Which action should the nurse take FIRST? -Notify the Health Care Provider -Prepare the patient for a STAT MRI -Perform a bedside swallow study -Perform a focused neuro assessment

Appear unhurried and confident during assessment

A nurse is caring for a patient who states feeling "very anxious" about the health assessment. What steps can the nurse take to make the patient more comfortable? Appear unhurried and confident during assessment Stay in room when patient is undressing to assist Ask the patient to change into examining gown Defer taking vital signs till the end, to allow time to become comfortable

Move each joint until there is resistance but no pain

A nurse is caring for an inactive client and assisting the client in performing range-of-motion (ROM) exercises. What care should the nurse take when performing range-of-motion exercises? -Use pillows and other positioning devices. -Move each joint until there is resistance but no pain. -Perform different movements with each extremity. -Change the pattern of exercises each time.

-Influenza -Pneumonia -Skin infection

A nurse is caring for an older adult client at a long-term health care facility. Which infections pose a risk to long-term care residents and older adult clients admitted to health care facilities? Select all that apply. -Influenza -HIV -Pneumonia -Skin infection -Chickenpox

Alarm-activating bracelet

A nurse is caring for an older adult client at risk of injury due to confusion. The client has a stable gait. Which method of restraining should the nurse use? -Locking lap tray chair -Vest restraint -Alarm-activating bracelet -Waist restraint

Standard precautions

A nurse is changing the bed linen of a client admitted to the health care facility. Which isolation precaution should the nurse follow? -Contact precautions -Droplet precautions -Standard precautions -Airborne precautions

bacteremia

A nurse is changing the soiled bed linens of an older adult client who has urinary incontinence and is hospitalized. The nurse monitors the client closely based on the understanding that this client is at greater risk for: -bacteremia. -otitis media. -impetigo. -gonorrhea.

"Mr. Koeppe, tell me what you do to take care of yourself."

A nurse is collecting information from a client with dementia. The client's daughter accompanies the client. Which statement by the nurse would recognize the client's value as an individual? -"Mr. Koeppe, tell me what you do to take care of yourself." -"Sarah, I have to go and read your father's old charts before we talk." -"Can you tell me how long your father has been this way?" -"Mr. Koeppe, I know you can't answer my questions, but it's okay."

"Mr. Koeppe, tell me what you do to take care of yourself."

A nurse is collecting information from a client with dementia. The client's daughter accompanies the client. Which statement by the nurse would recognize the client's value as an individual? -"Mr. Koeppe, tell me what you do to take care of yourself." -"Sarah, I have to go and read your father's old charts before we talk." -"Mr. Koeppe, I know you can't answer my questions, but it's okay." -"Can you tell me how long your father has been this way?"

"You are able to read at 20 ft (6 m) what a person with normal vision can read at 30 ft (9 m)."

A nurse is completing a vision exam with the Snellen eye chart and records the client's vision as 20/30 or 6/9. The client asks the nurse, "What does that mean?" How should the nurse respond? -"You are able to read at 20 ft (6 m) what a person with normal vision can read at 30 ft (9 m)." -"Your vision is better than average; you can read from 30 ft (9 m) what a person with normal vision can read from 20 ft (6 m)." -"Your vision in your right eye is slightly different than that of your left eye." -"Your vision is perfect; you can read the entire chart and you do not need glasses."

-placing a nightlight in the bathroom and the hallways -removal of clutter on the floor -moving the bedroom to the ground floor

A nurse is conducting a home assessment of a 90-year-old male client with a history of several minor strokes that have left him with a hemiplegic gait. The nurse is particularly concerned about falls. Which activities would help to prevent falls for this client? Select all that apply. -placing a nightlight in the bathroom and the hallways -installing hardwood floors -removal of clutter on the floor -moving the bedroom to the ground floor

Tympany

A nurse is conducting a physical examination and is percussing the gastric area of a client. What percussion tone is normally heard in this area? -Resonant -Flat -Dull -Tympany

humoral immunity

A nurse is developing a presentation for a local community group about infections and resistance to them. When describing acquired specific defenses, what would the nurse most likely include? -intact skin -gastric acid secretion -humoral immunity -phagocytosis of white blood cells

3+ pitting edema

A nurse is examining a client with cirrhosis of the liver for edema. The nurse notes that the indentation remains for several seconds and the skin swelling is obvious on inspection. How should the nurse quantify the severity of the finding? -1+ pitting edema -2+ pitting edema -5+ brawny edema -3+ pitting edema

helps to determine prescribed antibiotic therapy

A physician performs lumbar puncture and advises the nurse to send the obtained cerebrospinal fluid for Gram stains. The nurse understands that this type of testing is beneficial for which reason? -narrows the therapeutic range to avoid prolonged use -helps to determine prescribed antibiotic therapy -helps in reducing proliferation of multidrug-resistant organisms -permits selection of antibiotic concentration

Infected human → stool → vehicle transmission → gastrointestinal tract

A nurse is explaining the transmission of hepatitis A to a new public health nurse. Which of the following explanations of the chain of infection would be most correct? -Infected human → stool → vehicle transmission → gastrointestinal tract -Infected human → vector-borne transmission → blood → nonintact skin -Infected human → gastrointestinal tract → contact transmission → stool -Contact transmission → stool → gastrointestinal tract → infected human

The nurse details the client's response and the examination and treatment of the client after the incident.

A nurse is filing a safety event report for an older adult client who tripped and fell when getting out of bed. Which action exemplifies an accurate step of this process? -The nurse adds the information in the safety event report to the client health record. -The nurse calls the primary health care provider to fill out and sign the safety event report. -The nurse details the client's response and the examination and treatment of the client after the incident. -The nurse provides an opinion of the physical and mental condition of the client that may have precipitated the incident.

The nurse details the client's response and the examination and treatment of the client after the incident.

A nurse is filing a safety event report for an older adult client who tripped and fell when getting out of bed. Which action exemplifies an accurate step of this process? -The nurse details the client's response and the examination and treatment of the client after the incident. -The nurse calls the primary health care provider to fill out and sign the safety event report. -The nurse adds the information in the safety event report to the client health record. -The nurse provides an opinion of the physical and mental condition of the client that may have precipitated the incident.

Before and after dressing change, Using soap and water after removing gloves, Before medication administration

A nurse is finished changing a wound dressing for a patient on contact precautions for Clostridium difficile. Which of the following would be appropriate hand hygiene? Select all that apply: -Before and after dressing change -Using soap and water after removing gloves -Before medication administration -Using alcohol-based gel when leaving room

Wear gloves whenever entering the client's room

A nurse is in charge of care for a client who has MRSA. Which of the following is an accurate guideline for using transmission-based precautions when caring for this client? -Place the client in a private room that has monitored negative air pressure. -Keep visitors 3 feet (1 m) from the client. -Use respiratory protection when entering the room. -Wear gloves whenever entering the client's room.

Inspect the sterile kit for package integrity, contamination, or moisture.

A nurse is inserting an indwelling catheter into a female patient (a sterile procedure). Which is the most important first step in maintaining a sterile field? -Never turn your back to the sterile field. -Unfold sterile drape away from your body. -When adding sterile supplies, hold 10 to 12 inches above the field and allow them to drop. -Inspect the sterile kit for package integrity, contamination, or moisture.

Rubs

A nurse is listening to the lung sounds of a severely dehydrated client. The nurse hears sounds that are described as grating or leathery. What type of adventitious sounds are these? -Rubs -Crackles -Wheezes -Gurgles

with sterile forceps or hands wearing sterile gloves

A nurse is performing a sterile dressing change. If new sterile items or supplies are needed, how can they be added to the sterile field? -by carefully handling them with clean hands -by clean hands wearing clean latex gloves -with clean forceps that touch only the outermost part of the item -with sterile forceps or hands wearing sterile gloves

The client's pupils are black, equal in size, and round and smooth

A nurse is performing eye assessments at a community clinic. Which assessment would the nurse document as normal? -An older adult's pupils are pale and cloudy. -The client's pupils are black, equal in size, and round and smooth. -The client's eyes do not converge when the nurse moves a finger toward his nose. -The client's pupils dilate when looking at a near object and constrict when looking at a distant object.

"Let me tell you what I will be doing. It should not be painful."

A nurse is preparing a client for a physical assessment. The client appears anxious about the assessment. Which statement by the nurse would be most appropriate? -"Let me tell you what I will be doing. It should not be painful." -"This is nothing to worry about. I won't hurt you." -"Some of the examination may be painful, but I will be gentle." -"I have to do this, so just relax and it won't last long."

Facing away from the body

A nurse is preparing a sterile field and has removed the sterile drape from the outer wrapper. The nurse places the inner drape in the center of the work surface with the outer flap facing in which direction? -Facing toward the body -Facing away from the body -Toward the right side -Angled to the left side

intravenous antibiotic adminstration

A nurse is preparing an education plan for a client being discharged home after successful treatment for a wound infection. What would the nurse be least likely to include in the education plan? -vital sign monitoring -hand hygiene measures -signs and symptoms of infection -intravenous antibiotic adminstration

Position it with the inside facing up on a flat surface

A nurse is preparing to add a sterile solution to a sterile container on a sterile field. After opening the container, what would the nurse do with the cap? -Place it rim down on the corner of the sterile field. -Place it in the biohazard receptacle. -Hold it in the nondominant hand. -Position it with the inside facing up on a flat surface.

-gloves -stethoscope -pen light

A nurse is preparing to complete a basic physical assessment. Which supplies will the nurse gather before preparing the client? (Select all that apply.) -gloves -stethoscope -Snellen chart -pen light -ophthalmoscope

"What brings you here today?"

A nurse is preparing to conduct the health history for a client new to the clinic. Which question would the nurse likely ask first? -"Do you have any pain or discomfort?" -"How would you describe your health?" -"What brings you here today?" -"What are your usual activities each day?"

The nurse should record the incident in the client's medical record and fill out a safety event report separately

A nurse is preparing to file a safety event report after a client experienced a fall. Which statement is correct regarding the filing of a safety event report? -The nurse should include a note on the client's chart that mentions the report. -The nurse should await results of the x-ray before filing the report. -The nurse should make a copy of the safety event report and place it in the client's medical record. -The nurse should record the incident in the client's medical record and fill out a safety event report separately.

selecting an appropriate vein and establishing access aseptically

A nurse is preparing to insert a peripheral intravenous (IV) into a client who requires IV fluids. How can the nurse best demonstrate the skills that indicate the nurse meets the Quality and Safety Education for Nurses (QSEN) competency of safety? -knowing the assessment findings that indicate therapy has been successful selecting an -appropriate vein and establishing access aseptically -being aware of the signs of infiltration and other complications of therapy -having empathy for the client and recognizing that the procedure is painful

uses broad, open statements to communicate with the client

A nurse is preparing to interview a client as part of the assessment. The nurse demonstrates knowledge of communication skills when the nurse: -agrees with each of the client's statements. -attempts to write down everything the client says. -uses broad, open statements to communicate with the client. -reassures the client of good outcomes.

Area of active drainage

A nurse is preparing to obtain a specimen for an aerobic wound culture. The nurse would obtain the specimen from which area? -Deep into the cavity -Area of active drainage -Soiled dressing -Edge of the wound

Deep into the cavity

A nurse is preparing to obtain a specimen for an anaerobic wound culture. The nurse would obtain the specimen from which area? -Edge of the wound -Drainage on the dressing -Area of active drainage -Deep into the cavity

Limit own exposure to radiation to the minimum time

A nurse is preparing to provide care to a client who is receiving radiation therapy for cancer; the radiation source will be kept in the client's room. Which action would be most important for the nurse to do? -Ensure client is admitted to a semi-private room. -Limit own exposure to radiation to the minimum time. -Dispose of linens immediately upon exposure. -Wear an apron made of thick cotton material.

Position a friction-reducing sheet under the client

A nurse is preparing to turn a hospitalized client age 65 years. Which action is a recommended guideline for performing this skill? -Use back muscles to pull the client to the side. -Place the bed in its lowest position. -Position a nurse at the top and bottom of the bed. -Position a friction-reducing sheet under the client.

the client's ability to assist

A nurse is providing care for a client who has been newly admitted to the long-term care facility. What is the primary criterion for the nurse's decision whether to use a mechanized assistive device for transferring the client? -the client's age -the client's body weight -the client's cognitive status -the client's ability to assist

isometric

A nurse is providing care to a client who has a cast on his leg. While educating the client on how to perform quadriceps setting exercises to strengthen the quadriceps muscle, he asks, "What kind of exercise is this?" The nurse identifies this type of exercise as: -anaerobic. -isometric. -aerobic. -isotonic.

generation of a fever response

A nurse is reviewing laboratory test results and finds that a client's neutrophil levels are elevated. The nurse understands that these cells are important for: -generation of a fever response. -synthesis of immunoglobulins. -disposal of cellular debris. -antigen-antibody response.

3.2 mmol/L

A nurse is reviewing the laboratory test results of a client who is at high risk for septic shock. Which serum lactate level would the nurse identify as indicating sepsis? -1.4 mmol/L -3.2 mmol/L -2.6 mmol/L -0.8 mmol/L

Shave with the direction of hair growth in downward, short strokes

A nurse is shaving the facial hair of a client confined to bed. What is a recommended guideline for this procedure? -Shave with the direction of hair growth in downward, short strokes. -Apply shaving cream approximately 1 inch thick. -Fill bath basin with cool water. -Do not use aftershave or lotion on area shaved.

The nurse manually lifts a heavy client rather than using a mechanical lift.

A nurse is struggling to provide adequate care during an exceptionally busy shift. Which action is most likely a violation of Occupational Safety and Health Administration (OSHA) regulations? -The nurse manually lifts a heavy client rather than using a mechanical lift. -The nurse delegates administration of a client's oral medications to a licensed practical nurse. -The nurse flushes a client's intravenous line with 0.45% NaCl rather than 0.9% NaCl. -The nurse is unable to begin documentation until 3 hours into the shift.

The nurse manually lifts a heavy client rather than using a mechanical lift

A nurse is struggling to provide adequate care during an exceptionally busy shift. Which action is most likely a violation of Occupational Safety and Health Administration (OSHA) regulations? -The nurse manually lifts a heavy client rather than using a mechanical lift. -The nurse flushes a client's intravenous line with 0.45% NaCl rather than 0.9% NaCl. -The nurse is unable to begin documentation until 3 hours into the shift. -The nurse delegates administration of a client's oral medications to a licensed practical nurse.

back massage

A nurse is taking care of a client who needs a bed bath. Which action can the nurse delegate to an unlicensed assistive personnel (UAP)? -back massage -skin assessment during bed bath -assessment of sensation in lower extremities -wound care

Examine testicles for lumps monthly while showering.

A nurse is teaching a client about testicular self-examination. What should be included in the teaching? -Examine testicles for lumps monthly while showering. -Squeeze each testicle gently feeling for lumps twice a month. -Check the testes weekly for lumps while laying down in bed. -Visualize the testes in the mirror looking for lumps monthly.

"According to the American and Canadian Cancer Societies, your first mammogram should be done at age 40 and then yearly after that."

A nurse is teaching a young female client about breast cancer prevention. The client asks at what age does she need to begin having mammograms. What is the nurse's best response? -"Why do you want to know? Do you have a history of breast or ovarian cancer in your family?" -"Don't worry about that yet; you are still young. You will not need a mammogram until you are in your 40s." -"Your physician will decide when it is best for you to begin having mammograms based on your family history." -"According to the American and Canadian Cancer Societies, your first mammogram should be done at age 40 and then yearly after that."

Reflexes

A nurse is testing the function of the spinal cord of a client who presents in the emergency department following a motorcycle accident. What would be the focus of this assessment? -Sensory abilities -Balance and gait -Reflexes -Motor ability

using carbon monoxide detectors and alarms

A nurse is working as an industrial nurse. Which activity would the nurse suggest that the employers adopt to prevent carbon monoxide (CO) inhalation by the workers? -educating the workers about signs of the presence of CO gas -ensuring a good ventilation system at the work place -using carbon monoxide detectors and alarms -keeping the resuscitation equipment ready at hand

"If I experience pain when I'm exercising, I should stop."

A nurse is working with a female client with osteoporosis on an exercise program. The nurse instructs the client to increase her tolerance gradually. The nurse determines that the education was effective when the client states: -"Strength training will be of little benefit to me." -"My initial goal is to exercise every day of the week." -"If I experience pain when I'm exercising, I should stop." -"I need to avoid weight-bearing exercises."

Ambulation

A nurse is working with an 82-year-old man following gallbladder surgery. He is n.p.o. and has IV access in his hand. He also has a Foley catheter in place. He is able to ambulate with the aid of a walker. What does not lower this client's immunity? -Surgical incision -Ambulation -IV access -Foley catheter

This action is a violation of Occupational Safety and Health Administration (OSHA) regulations

A nurse manager has received an incident report that describes intravenous tubing containing chemotherapeutics being found in the regular trash, rather than in the specially designated receptacle. Which statement accurately describes an aspect of this situation? -This action is a serious breach of protocol that could possibly result in criminal charges. -This action is a violation of Occupational Safety and Health Administration (OSHA) regulations. -The hospital where this occurred is likely to lose Centers for Medicare & Medicaid Services (CMS) funding. -A root cause analysis will determine which staff member committed this error.

The nurse demonstrated reckless behavior by not admitting the error

A nurse mistakenly gave a client an immediate dose of an opioid, rather than the extended release form of the drug. The client developed respiratory depression that required resuscitation. The nurse did not admit to the error until forced to weeks later by persistent rumors among colleagues. Which statement about this nurse's actions is most accurate? -The nurse demonstrated at-risk behavior by covering up the error. -The nurse demonstrated human error by the attempts to cover up the mistake. -The nurse demonstrated reckless behavior by administering the wrong version of the medication. -The nurse demonstrated reckless behavior by not admitting the error.

Over the client's thigh

A nurse needs to measure the blood pressure of a client who has just undergone a bilateral mastectomy. How should the nurse measure the blood pressure? -Radial artery -Over the client's thigh -Over the lower arm -Brachial artery

Pour and discard a small amount of the solution

A nurse needs to visit the intensive care unit to administer an enema to a client. Which step should the nurse take when using the sterile solution located at the entrance to the intensive care unit? -Loosen the cap or the seal on the bottle. -Pour and discard a small amount of the solution. -Clean the nozzle area with a damp cloth. -Hold the container from the top.

Pulse amplitude

A nurse palpates the pulse of a client and documents the following: 6/6/12 pulse 85 and regular, +3, and equal in radial, popliteal, and dorsalis pedis. What does the number +3 represent? -Pulse amplitude -Pulse rate -Pulse deficit -Pulse rhythm

Time-lapse

A nurse performs an assessment of a client in a long-term care facility and records baseline data. The nurse reassesses the client a month later and makes revisions in the plan of care. What type of assessment is the second assessment? -Emergency -Comprehensive -Focused -Time-lapse

otoscope

A nurse practitioner is preparing to examine a child with a suspected otitis media. Which instrument will be required? -pen light -pphthalmoscope -otoscope -sphygmomanometer

completing an incident report describing this near miss

A nurse prepared a client's medication, brought it to the client's bedside and then realized at the last minute that the medication was for another client of similar age and appearance. Follow-up to this event should include: -completing an incident report describing this near miss. -documenting this latent error and reporting it to relevant state agencies. -a suspension for the nurse, pending the completion of remedial education. -revising the procedures by which medications are distributed on the unit.

Pull the fire alarm lever

A nurse responds to the call bell and finds another nurse evacuating the client from the room, which has caught fire. Which action should the nurse take? -Confine the fire. -Extinguish the fire. -Pull the fire alarm lever. -Evacuate the unit.

area rugs kept on the stairs without carpet

A nurse visits an older adult client at home and assesses the safety of the client's environment. Which article can be a threat to the client's safety? -area rugs kept on the stairs without carpet -carpet on the floor of the living room -skid-resistant small area rugs on the floor -a laundry bag at the corner of the room

the reprimand has been deemed appropriate and relevant to the nurse's actions.

A nurse who is a recent graduate has been formally reprimanded after administering a medication to the wrong client. This reprimand would be considered to be consistent with a just culture if: -the nurse received ample support from colleagues during the discipline process. -the needs of the client were prioritized over the needs of the nurse. -the reprimand has been deemed appropriate and relevant to the nurse's actions. -the reprimand was legally documented.

decreased saliva production

A nurse working in a clinic for older adults is providing care to a client receiving treatment for xerostomia. The nurse interprets this condition as: -use of dentures. -receding gums. -decreased saliva production. -periodontal disease.

chill

A nursing instructor is describing the phases of a febrile episode. What would the instructor describe as happening first?

Monocytes, T lymphocytes

A nursing instructor is preparing a class about the different types of white blood cells. what would the instructor include as agranulocytes?

Neutrophils, basophils, and eosinophils

A nursing instructor is preparing a class about the different types of white blood cells. what would the instructor include as granulocytes

Nursing staff

A nursing student is discussing assessment findings of an assigned client with the instructor. The instructor determines that the student needs additional assistance and review when the student identifies which as objective data? -Physical assessment -Nursing staff -Health records -Measurement values

reaches down to the bed to pick up a sterile drape

A nursing student is donning sterile gloves to perform routine tracheostomy care for a client. Which behavior by the student would require immediate intervention from the instructor? -reaches down to the bed to pick up a sterile drape -picks up the glove at the folded edge with the thumb and forefinger -washes hands for 20 seconds with soap and water -stretches the glove over the hand without touching the unsterile area

Inflate the cuff to 30 mm Hg above reading where brachial pulse disappeared

A nursing student is manually taking the client's blood pressure. Which step will demonstrate the correct way of inflating the blood pressure cuff? -Elevate arm above heart level before inflating the cuff. -Fully inflate cuff for about 1 minute. -Place cuff 8 cm above the elbow. -Inflate the cuff to 30 mm Hg above reading where brachial pulse disappeared.

adjusts the water temperature to be hot

A nursing student is preparing to return demonstrate the skill of handwashing. Which action would indicate that the student needs additional education? -avoids leaning against the wet sink -adjusts the water temperature to be hot -pushes watch to about 4 in (10 cm) above the wrist -removes jewelry except for plain wedding band

sepsis

A patient admitted with pneumonia appears to be worsening. The patient is newly confused and has decreased blood pressure and increased heart and respiratory rate. Lab work also shows a very high WBC count. The nurse should be suspicious of what condition? -Opportunistic infection -Nosocomial infection -Neutropenia -Sepsis

Sepsis

A patient diagnosed with pneumonia is exhibiting an elevated temperature, high white blood count, and low blood pressure with increased heart rate. Which of the following would describe the patient's condition? -Sepsis -Infectious disease -Opportunistic infection -Nosocomial infection

opportunistic infection

A patient is experiencing new onset of diarrhea and abdominal pain following a course of chemotherapy. Which of the following best describes this illness? -Opportunistic infection -Colonization -Neutropenia -Immunodeficiency

droplet

A patient is identified as having MRSA in his sputum. Which of the following transmission routes would be most appropriate? -Contact -Vehicle -Airborne -Droplet

false

A person's core body temperature is highest in the early morning and lowest in the late afternoon. True False

Tuning fork

A school nurse is preparing to test the auditory function of grade school students. What equipment will be needed for this examination? -Speculum -Percussion hammer -Ophthalmoscope -Tuning fork

identifying strategies for preventing a repeat of this event

A sentinel event involving a client's suicide attempt on a hospital unit is being analyzed by the hospital's quality and safety committee. The committee should prioritize what consideration when performing this analysis? -identifying the hospital's and staff members' liability in the case -identifying strategies for preventing a repeat of this event -apportioning blame fairly and providing supplementary education -mapping the chain of command that existed immediately before the event

eliminating interruptions to nurses while they are preparing medications

A series of medication errors have prompted a reevaluation of the drug administration practices on a unit. Which of the following strategies is most likely to prevent errors and enhance safety? -ensuring that no nurse is responsible for more than four clients -banning the practice of nurses' delegating medication administration to other nurses -eliminating interruptions to nurses while they are preparing medications -working with the pharmacy to avoid the use of medications that have similar names

Normal body temperature

A student is reading the medical record of an assigned client and notes the client has been afebrile for the past 12 hours. What does the term "afebrile" indicate? -Fluctuating body temperature -Decreased body temperature -Normal body temperature -Increased body temperature

"To prevent the legs from rotating outward."

A student nurse asks the nurse what trochanter rolls are used for when providing client care. What is the appropriate nursing response? -"To prevent the legs from rotating outward." -"To preserve the client's functional ability to grasp and pick up objects." -"To avoid contractures." -"To prevent foot drop."

Ask the instructor or a staff nurse to take the pulse

A student takes an adult client's pulse and counts 20 beats/min. Knowing this is not the normal range for an adult pulse, what should the student do next? -Discuss this finding during postconference with other students. -Wait 4 hours and take the client's pulse again. -Record the pulse rate on the appropriate vital signs sheet in the chart. -Ask the instructor or a staff nurse to take the pulse.

hyperpyrexia

A temperature greater than 40.5°C is referred to as?

The immune system is not fully operational until 6 months of age. Formula does not provide the infant with antibodies. Newborns have immature thermoregulation and may not become febrile.

A triage nurse receives a call from a mother worried about her formula-fed, 1-month-old infant. The infant is lethargic and not feeding regularly but is afebrile. The nurse tells the mother to bring the infant to the physician's office. Which of the following are relevant for this newborn's immunity and illness? Select all that apply: -The immune system is not fully operational until 6 months of age. -Formula does not provide the infant with antibodies. -Newborns have immature thermoregulation and may not become febrile. -Vaccinations will cover periods until the infant develops immunocompetence.

angina

A type of chest pain caused by reduced blood flow to the heart.

-Hepatitis B -HIV -Hepatitis C

A veteran nurse is working with a new graduate nurse. The graduate nurse states that she was exposed to a client's blood and that she was not wearing any PPE. Which would be considered significant blood exposures by occupational health? Select all that apply. -Hepatitis B -HIV -Tuberculosis -Hepatitis C

Decreased cardiac output

A weak, thready pulse found after the nurse palpates peripheral pulses may indicate which condition? -Decreased cardiac output -Hypertension and circulatory overload -Impaired circulation -Inflammation of a vein

viruses

acute living microorganism composed of particles of nucleic acid and protein that are often membrane bound. (reproduce inside living things)

Hand hygiene must be performed after contact with inanimate objects near the client

An experienced nurse is educating a student nurse on the proper use of hand hygiene. What is an accurate guideline that should be discussed? -The use of hand hygiene eliminates the need for gloves. -Hand hygiene must be performed after contact with inanimate objects near the client. -Hand lotions should not be used after hand hygiene. -The use of gloves eliminates the need for hand hygiene.

Explain to the client how the nurse will assist her

An obstetrics nurse is preparing to help a client up from her bed and to the bathroom 3 hours after the woman gave birth. Which action should the nurse perform first? -Have the client stand for 30 seconds prior to walking. -Enlist the assistance of another nurse or the physiotherapist. -Explain to the client how the nurse will assist her. -Position a walker in front of the client to provide stability.

"Is there anything else we should know in order to care for you better?"

After conducting the initial assessment of a new resident of a long-term care facility, the nurse is preparing to terminate the interview. Which question is the most appropriate conclusion to the interview? -"Is there anything else we should know in order to care for you better?" -"What practices have you found especially helpful in other settings?" -"What are your expectations from us and from yourself in your care?" -"What do you envision for your care while you're here at the facility?"

"Health care records are primarily used for communication among nurses and physicians."

After educating a group of nursing students about the health care record and its purposes, the instructor determines that the group needs additional instruction when the students state: -"Health care records are primarily used for communication among nurses and physicians." -"The health care record provides valuable information about a client's assessment." - "The record provides a means for decisions about reimbursement for care." -"The health care record can serve as a resource for conducting research."

decreased cellular immunity

After educating students about changes in the immune system and risk for infection as people age, the instructor determines that the education was successful when the students identify: -increased humoral immunity response. -decreased susceptibility to infection. -increased effectiveness of phagocytosis. -decreased cellular immunity.

acute phase

After explaining to students about the progression of infection, an instructor determines that the education was successful when specific symptoms and often laboratory analysis can identify the disease -incubation period -acute phase -prodromal phase -communicable period

incubation period

After explaining to students about the progression of infection, an instructor determines that the education was successful when the students identify which period as the time between the pathogen's entrance into the host and the appearance of symptoms -incubation period -active phase -prodromal phase -communicable period

communicable period

After explaining to students about the progression of infection, an instructor determines that the education was successful when the students identify which period as the time during which a disease can be passed from one person to another? -incubation period -active phase -prodromal phase -communicable period

prodromal period

After explaining to students about the progression of infection, an instructor determines that the education was successful when the students identify which period as the time during which a disease is characterized by nonspecific symptoms. -incubation period -active phase -prodromal phase -communicable period

Objective

After performing the admission assessment on an older adult client, the nurse documents the following, "Client observed fidgeting with covers; facial grimacing when turning from side to side." This documentation is an example of which type of data? -Subjective -Physical -Unreliable -Objective

1.5 in (3.75 cm)

After preparation, the nurse inserts a rectal thermometer into an adult client's rectum. To ensure an accurate reading, the nurse inserts the thermometer to which depth? -1 in (2.5 cm) -2 in (5 cm) -1.5 in (3.75 cm) -0.5 in (1.25 cm)

Eosinophils

After teaching a group of nursing students about the function of the various white blood cells, the instructor determines that the teaching was successful when the students identify which cell as being involved with allergic reactions? -Eosinophils -Neutrophils -Basophils -Monocytes

activity of the glands in the skin lessens

An 80-year-old woman tells the nurse that she just itches all the time and her skin seems very dry. How do these symptoms relate to aging skin? -activity of the glands in the skin lessens -the symptoms are unrelated to aging skin -skin gland activity increases, leading to acne -the symptoms are indicators of a disease

Collaborate with the nurse to identify any supplementary education that would be beneficial

An experienced nurse has a reputation for being conscientious and caring, so the nurse is shocked and embarrassed to have committed a medication error for the first time in her career. The nurse's supervisor should use what approach when responding to this event? -Temporarily assign the nurse a mentor so that her confidence can be reestablished. -Collaborate with the nurse to identify any supplementary education that would be beneficial. -Reassure the nurse that future errors are highly unlikely, given her strong work history. -Allow the nurse to follow up the event independently, knowing that she possesses the necessary knowledge and experience.

Focused assessment

An older adult male with a history of benign prostatic hyperplasia (BPH) presents to the emergency room with reports of urinary retention. The nurse collects data related to the client's voiding patterns, weight gain, fluid intake, urine volume in the bladder, and level of suprapubic discomfort. What type of assessment is the nurse performing? -Emergency assessment -Initial assessment -Focused assessment -Time-lapse assessment

Focused assessment

An older adult male with a history of benign prostatic hyperplasia (BPH) presents to the emergency room with reports of urinary retention. The nurse collects data related to the client's voiding patterns, weight gain, fluid intake, urine volume in the bladder, and level of suprapubic discomfort. What type of assessment is the nurse performing? -Time-lapse assessment -Focused assessment -Emergency assessment -Initial assessment

Fractured hip

An older adult woman in a long-term care facility has fallen and sustained several injuries. Which of her injuries would be the most serious fall-related injury? -Fractured ulna -Fractured hip -Lacerated lip -Thigh contusion

Recommended changes are consistently implemented at the point of care

Analysis is being conducted of a sentinel event that result in injury to a client. When putting forward recommendations to prevent future similar events, the safety committee should prioritize which of the following? -Proposed changes are based on current peer-reviewed evidence and clinical expertise. -Changes in practice are reflective of clients' and families' preferences. -The role of nurses in promoting overall safety in the hospital is acknowledged. -Recommended changes are consistently implemented at the point of care.

antiseptic

agent that stops or slows the growth of microorganism on living tissue, commonly used for hand washing skin preparation and wound packing or irrigation

The blood pressure increases

As adults age, the walls of their arterioles become less elastic, increasing resistance and decreasing compliance. How does this affect the blood pressure? -The blood pressure does not change. -The blood pressure decreases. -The blood pressure is erratic. -The blood pressure increases.

Greater than 40.5°C

Assessment of a client's temperature reveals hyperpyrexia. The nurse interprets this as indicating that the client's temperature is most likely: -Greater than 40.5°C -Above 38.2°C -Between 35°C and 36.8°C -Between 37.1°C and 38.2°C

virulence

refers to the degree of damage caused by a microbe to its host. The pathogenicity of an organism - its ability to cause disease

xerostomia

Dry mouth

-Competence -Respect for client -Caring -Professionalism

Which quality does the nursing student identify as being helpful in inviting the confidence of clients when first working with them? Select all that apply. -Number of years in profession -Competence -Respect for client -Caring -Professionalism

anticoagulant

During a safety huddle, medication safety has been reviewed by the nurse educator on a unit. A nurse on the unit would be implementing the Joint Commission 2015 Hospital National Patient Safety Goals by being particularly vigilant when administering a client's: -anticoagulant. -potassium-wasting diuretic. -opioid analgesic. -oral antihyperglycemic.

true

During measurement of a rectal temperature, the thermometer probe should be inserted about 1.5 inches (3.8 cm) in an adult and 0.5 inches (1.3 cm) in an infant. True False

inversion and eversion of the ankle

During range-of-motion exercises, the nurse turns the sole of a client's foot toward the midline and then turns the sole of the foot outward. Which type of movement is this nurse promoting by these actions? -dorsiflexion and plantar flexion of the ankle -inversion and eversion of the ankle -flexion and extension of the ankle -internal and external rotation of the ankle

Stand behind the client and palpate the sides of the trachea

During the admission assessment of a new client, the nurse is preparing to assess the client's thyroid gland. How should the nurse perform this assessment? -Lightly percuss slightly off midline over the client's trachea. -Observe the midline of the client's neck while asking him to bear down. -Auscultate over the client's trachea while asking the client to hold his breath. -Stand behind the client and palpate the sides of the trachea.

"Your pulse is 80 beats per minute, which is within normal range."

During the assessment the nurse provides the patient with brief periods of explanation and teaching. Which statement made by the nurse is MOST appropriate? - "Your atrial dysrhythmias are under control." - "You have +3 pitting edema and mild varicosities." - "Your pulse is 80 beats per minute, which is within normal range." -"I am using my stethoscope to listen for crackles and wheezes."

body systems

During the initial assessment of a newly admitted client, the nurse has clustered the client's range of motion (ROM) with his gait, his bowel sounds with his usual elimination pattern, and his chest sounds with his respiratory rate. The nurse is most likely organizing assessment data according to: -body systems. -human needs. -functional health patterns. -human response patterns.

Ask the client if it is okay to interview her husband for the answers to the interview questions

During the nursing examination, the nurse notices that the client, an older adult female, becomes very tired, but there are still questions that need to be addressed in order to have data for planning care. Which action would be most appropriate in this situation? -Ask the client if it is okay to interview her husband for the answers to the interview questions. -Ask the client to wake up and try to answer the interview questions. -Wait until the next day to obtain the answers to the interview questions. -Ask the client's husband to come in and answer the interview questions.

Ask the client if it is okay to interview her husband for the answers to the interview questions

During the nursing examination, the nurse notices that the client, an older adult female, becomes very tired, but there are still questions that need to be addressed in order to have data for planning care. Which action would be most appropriate in this situation? -Ask the client to wake up and try to answer the interview questions. -Wait until the next day to obtain the answers to the interview questions. -Ask the client's husband to come in and answer the interview questions. -Ask the client if it is okay to interview her husband for the answers to the interview questions.

helps prevent overlooking some aspect of data collection reduces the number of position changes required of the client takes less time because the nurse doesn't have to constantly move around the client

During the physical assessment of a client, the nurse uses the head-to-toe approach. What are the advantages of this approach? Select all that apply. -takes less time because the nurse doesn't have to constantly move around the client -makes the problem easily identifiable because the findings tend to be clustered -examines the same areas of the body several times before the assessment is complete -reduces the number of position changes required of the client -helps prevent overlooking some aspect of data collection

cerumen

Earwax

skin is dry and intact

Every 2 hours, the nurse turns and repositions the client who is experiencing frequent diarrhea. This action supports, among other things, infection prevention. Which assessment indicates that there is a positive outcome from this nursing care? -blanching over elbow area noted -slight bleeding noted while old dressing is removed -skin is dry and intact -redness size over sacral area is with minimal increase

Applying restraints to patients at risk for falls

Fall prevention is done on multiple different levels by nurses in the hospital. Which of the following would not be an example of fall prevention done by nursing? -Reminding the patient to use the call light for assistance when getting out of bed -Placing the bed mattress on the floor to prevent injury -Developing an organizational policy for hourly rounding schedules to be performed on each unit to monitor patient needs and comfort level -Applying restraints to patients at risk for falls

chill phase

Gooseflesh is evident during what phase of the fever?

Pneumonia, Urinary tract infections, SSIs, Central IV line infections

HAIs are gaining attention as preventable conditions among hospitalized patients. Which of the following include common HAIs? Select all that apply: -Pneumonia -Urinary tract infections -SSIs -Central IV line infections

alopecia

Hair loss

pediculosis

Infestation with lice

"Client states, 'I don't see the point in trying anymore.'"

How should a nurse best document the assessment findings that have caused her to suspect that a client is depressed following his below-the-knee amputation? -"Client makes statements indicating a loss of hope." -"Client states, 'I don't see the point in trying anymore.'" -"Client is demonstrating signs and symptoms of depression." - "Client states that his rehabilitation will be unsuccessful."

Ask the client to focus on an object as it is brought closer to the nose

How would a nurse assess a client for pupillary accommodation? -Ask the client to focus on an object as it is brought closer to the nose. -Ask the client to read the smallest possible line of letters on the Snellen chart. -Using an ophthalmoscope, check the red reflex. -Ask the client to focus on a finger and move the client's eyes through the six cardinal positions of gaze.

c & d

If a sentinel event occurred in your hospital facility related to a medication error, to which regulatory body/bodies is the hospital required to report the event? -U.S. Department of Labor's OSHA -National Academy of Science's IOM -The Joint Commission Center for Transforming Healthcare -State Department of Health and/or other health agencies -a & d -a & c -c & d

5+ brawny

In the case of this type of edema, there is no pitting; tissue palpates as firm or hard, and the skin surface appears shiny, warm, and moist.

dysphagia

Inability to swallow

fiber

Increased intake of this would lead to decreased heat production

a harsh, inspiratory sound that may be compared to crowing

It is very important to assess for the quality of someone's respirations as well as describe what is heard with auscultation. Which describes stridor? -high-pitched musical sound -discontinuous popping sounds -a harsh, inspiratory sound that may be compared to crowing -respirations that require excessive effort

renal calculi (kidney stones)

Laboratory results for a client on prolonged bed rest include a high level of urinary calcium. What risk does this pose for the client? -renal calculi (kidney stones) -imbalanced intake/output -increased urinary output -Urinary calcium is not a concern.

Much hospital waste is considered a biohazard, but much other waste can be conventionally recycled.

Local media reports have focused on the large amounts of waste that are produced by a local hospital, and there is public pressure to reduce waste. What principle should guide the hospital's waste-reduction efforts? -Hospital waste can be recycled on-site, but the costs associated with this are prohibitive. -Much hospital waste is considered a biohazard, but much other waste can be conventionally recycled. -The technology does not yet exist for sorting hospital waste safely and efficiently. -Hospital waste is distinctly different from other waste because it contains blood and body fluids.

Pediculosis

is an infestation of lice

220

Maximum heart rate is calculated by subtracting the client's age from?

Position Mr. Ames' bed closer to the nurses' station and perform an assessment

Mr. Ames, age 84, has just been admitted to the hospital for the treatment of pneumonia. In addition to this diagnosis, Mr. Ames also has stage II Alzheimer's disease and is disoriented to place and time. As the night has progressed, he has become increasingly agitated, pulling out his intravenous catheter and wandering throughout the unit. He has become more agitated as the nurses have attempted to reorient and redirect him. Which intervention should the nurses perform? -Resettle Mr. Ames in his bed and raise the side rails. -Call a "code gray" in order to call hospital security to deal with Mr. Ames. -Position Mr. Ames' bed closer to the nurses' station and perform an assessment. -Apply a restraint vest to Mr. Ames once he has been returned to bed.

Investigate the possibility of discontinuing his catheter

Mr. Geno is an 87-year-old client in a nursing home who keeps trying to get out of bed to use the bathroom, despite having a catheter in place. Which intervention would help keep this client safe and could be used as an alternative to restraints? -Ask the family to limit visits to the morning hours when the client is calmest. -Remove personal items, such as plants, books, and radios, from the environment. -Investigate the possibility of discontinuing his catheter. -Reduce daily exercise to promote rest and relaxation.

Implementing a system to sort recyclables from waste that contains toxins or body fluids.

Municipal authorities have requested that the local hospital become more environmentally responsible. Which action best promotes environmental sustainability in a hospital setting? -Replacing washable bed linens with single-use sheets in order to save water. -Implementing a system to sort recyclables from waste that contains toxins or body fluids. -Obtaining an autoclave so disposable equipment can be safely sterilized for reuse. -Ensuring that all waste is thoroughly and hygienically incinerated on-site.

1,000

Neutropenia is present when the absolute neutrophil count (ANC) falls to fewer than _____cells/mm3

Color, moisture, and temperature of the skin

Of the following data, what type would be collected during a physical assessment? -Foods eaten that cause nausea -Type, amount, and duration of pain -Specific allergies resulting in itching -Color, moisture, and temperature of the skin

Color, moisture, and temperature of the skin

Of the following data, what type would be collected during a physical assessment? -Specific allergies resulting in itching -Color, moisture, and temperature of the skin -Type, amount, and duration of pain -Foods eaten that cause nausea

Decrease in sensory-perceptual function and cognitive judgment, Impaired thermoregulation, Medication side effects

Older adults may be at increased risk of injury related to physiologic factors. Which of the following are relevant risk factors unique to this population? Select all that apply: -Decrease in sensory -perceptual function and cognitive judgment -Alcohol impairment -Impaired thermoregulation -Medication side effects

"These results indicate that you are colonized with MRSA."

On a preoperative surgical unit, as a standard of care, all clients are swabbed for methicillin-resistant Staphylococcus aureus (MRSA). Prior to his surgery, a nurse notes that a specific client's results have come back positive. The client asks the nurse what this means. What is the nurse's best response? -"These results indicate that you are infected with MRSA." -"These results indicate that you are colonized with MRSA." -"These results indicate that you are contaminated with MRSA." -"Two positive tests are required before results can be confirmed."

1 hour

Owen is a 15-year-old client who is waking up postoperatively. He became combative and tried to strangle one of the nurses. A support team was called and 4-point restraints were applied in this emergent situation. How soon does a licensed provider need to assess the client and place the restraint order? -1 hour -15 minutes -30 minutes -4 hours

lowest drug level

Serum trough levels is also known as?

"These barriers help to prevent the transmission of infection."

Several family members are visiting a client with an antibiotic-resistant infection who has been placed on contact precautions. When the nurse teaches the visitors about wearing gloves and gown, a family member states, "I don't want to wear those. I can't catch anything just by holding my loved one's hand." What is the appropriate nursing response? -"You may choose to wear the gown, or the gloves." -"I understand; wearing these items is optional." -"Contact precautions discourage spreading microorganisms through the air." -"These barriers help to prevent the transmission of infection."

decrease in the pulse rate

Stimulation of the parasympathetic nervous system results in a?

serology tests

While reviewing a client's medical record, a nurse notes the different laboratory tests results of the client. What would the nurse identify as the direct indication of an antigen-antibody response? -serology tests -erythrocyte sedimentation rate -white blood cell (WBC) count -lactate level

A client suffers a hip fracture after falling off of a commode.

The Centers for Medicare & Medicaid Services (CMS) is unlikely to reimburse a hospital for treatment of which adverse event? -A client experiences severe blistering as a result of an adverse drug reaction. -A client suffers a hip fracture after falling off of a commode. -A client experiences respiratory depression after receiving an opioid. -A client suffers a tear to her vaginal wall during a vaginal birth.

Rapid and deep

The arterial blood gases for a client in shock demonstrate increased carbon dioxide and decreased oxygen. What type of respirations would the nurse expect to assess based on these findings? -Noisy and difficult -Shallow and slow -Rapid and deep -Absent and infrequent

Client strengths and assets can be identified

The assessment phase of the nursing process has to be well organized to prevent omission of pertinent information. What is an advantage of using the functional health framework? -it focuses on the client's major anatomic systems. -the nurse can systematically examine every part of the body. -Client strengths and assets can be identified. -It allows the nurse to collect data about the past and present condition of each organ or body system.

client does not yet show signs and symptoms

The nurse is caring for a patient that has a colonized infection. What assessment findings does the nurse anticipate? -oral temperature of 101° F -active periods of nausea, vomiting, and diarrhea -client does not yet show signs and symptoms -reports feeling better because the infection is resolving

Contact

The client is an employee on the medical unit at the local children's hospital. The nurse is an occupational health nurse educating the client on various routes of exposure. The nurse knows that as a hospital employee, the client is most susceptible to infection by what mode of transmission? -Vehicle -Contact -Airborne -Droplet

Ask the client to demonstrate self-blood pressure assessment

The client is self-monitoring blood pressure at home and reports that every reading is 150/90 mm Hg. What is the priority nursing intervention? -Report readings to primary care provider. -Provide the client with a larger blood pressure cuff. -Recommend lower sodium in the client's diet. -Ask the client to demonstrate self-blood pressure assessment.

perform an initial history specific to substance use

The client reports, "I have a few drinks with friends every week." Which nursing action exemplifies using a focused assessment? -perform an initial history specific to substance use -interview friends to ascertain if client drinks more than what is reported -obtaining data regarding amount and frequency of drinking -ask the client to discuss social functioning

florence nightingale

is responsible for the basic principles of good health care

crisis phase of fever

The client will have profuse diaphoresis and reduced shivering during the what phase of the fever

"Would you consider using a digital thermometer?"

The home care nurse notices that the client only has a glass thermometer. What is the best response by the nurse? -"You need to throw that away, it is dangerous." -"Let's clean this off with hot water before use." -"Would you consider using a digital thermometer?" -"Why are you using this thermometer?"

on top of the foot

The nurse is assessing a client's pedal pulse. The nurse would palpate at which area? -on top of the foot -behind the knee -level of the fifth intercostal space -behind bony protuberance of the inner ankle

Reassess temperature after 1 hour and document results in the chart.

The nurse administered an antipyretic drug to a client with high-grade fever of 101.4°F (38.6°C). Which intervention should the nurse perform next? -Require the client to drink 8-10 glasses of water to avoid dehydration. -Reassess temperature after 1 hour and document results in the chart. -Encourage the client to ambulate in the room to improve circulation. -Ask the client to use his incentive spirometer 5 times.

"Alcohol based hand rub provides the greatest reduction in microbial counts on the skin."

The nurse applies an alcohol-based hand rub upon entering the client's room. The client becomes upset stating, "You did not wash your hands!" Which response by the nurse is most appropriate? -"Alcohol based hand rub provides the greatest reduction in microbial counts on the skin." -"We only wash our hands when they are visibly soiled." -"Washing the hands with soap and water is not necessary." -"I won't be touching you, so using the alcohol hand rub is the quickest method to perform hand hygiene."

applying a blanket

The nurse assesses that a client is shivering. Which intervention is most appropriate to prevent further stress on the body? -raising the room temperature -providing warm fluids -applying a cooling blanket -applying a blanket

An 80-year-old woman

The nurse caring for clients at an outpatient clinic determines that which client is at greatest risk for infection? -An 18-month-old infant -An 80-year-old woman -A 12-year-old girl -A 2-year-old toddler

"Wearing an N95 respirator is critical when I care for clients in droplet precautions."

The nurse educator is reminding a group of new nurses about precautions. Which statement by a new nurse requires further teaching by the nurse educator? -"Masks, gloves, and gowns should be used to protect from infectious agents." -"Wearing an N95 respirator is critical when I care for clients in droplet precautions." -"I will always wash my hands thoroughly and often." -"It is important to refrain from recapping needles."

culture and sensitivity

The nurse has admitted a new client to the unit. This client has an open draining sore on the leg. Which diagnostic test would the nurse anticipate being ordered? -culture and sensitivity -platelet count -urinalysis -sputum culture

Flexion

The nurse has asked the client to grasp his overbed trapeze and pull his torso up off the surface of the bed. What movement will the client perform with his arms? -Dorsiflexion -Abduction -Flexion -Adduction

assisting the client to put on slippers prior to ambulation

The nurse has delegated several parts of basic care for a client who is a fall risk to an unlicensed assistive personnel (UAP) member. Which UAP action requires nursing intervention? -reminding the client to sit on the bed for a few moments before standing -assisting the client to put on slippers prior to ambulation -assuring that there is a clear path between the bathroom and bed -placing the bed into the lowest setting

Apply an allergy-alert identification bracelet on the client

The nurse has just admitted a client with a latex allergy to the medical-surgical nursing floor. Which is the priority nursing intervention? -Teach client to wear Medic-Alert bracelet. -Apply an allergy-alert identification bracelet on the client. -Notify the interdisciplinary healthcare team to use nonlatex equipment. -Flag the room door

Repeat or read back the order

The nurse has received a medication order over the telephone from a provider. What is the next appropriate nursing action? -Identify the client by last name and date of birth. -Repeat or read back the order. -Prepare the medication for administration. -Document the order in the electronic health record (EHR).

Deep tendon reflexes

The nurse includes a percussion hammer in the assembled equipment to be used during a client's physical examination. The percussion hammer will be used to assess which of the following? -Vibratory perception -Tympanic membrane function -Auditory function -Deep tendon reflexes

Wheezing on auscultation

The nurse is assessing a 4-year-old child who has multiple bee stings. Which assessment finding would require immediate action by the nurse? -Crying with burning pain -Heart rate of 100 -Erythema at sting site -Wheezing on auscultation

nits from a lice infestation

The nurse is assessing a child brought to the clinic with severe itching of the scalp and white patches on the hair follicles. What would the nurse look for when beginning the examination? -tinea capitis -alopecia -nits from a lice infestation -seborrheic dermatitis

predisposition to renal calculi

The nurse is assessing a client who is bedridden. For which condition would the nurse consider this client to be at risk? -increase in the movement of secretions in the respiratory tract -increased metabolic rate -increase in circulating fibrinolysin -predisposition to renal calculi

Skin warm and flushed

The nurse is assessing a client with an elevated temperature. Which of the following would lead the nurse to determine that the client is in the fever phase? -Skin warm and flushed -Reduced shivering -Evidence of gooseflesh -Profuse diaphoresis

Ask the client to open and close the fist after cuff inflation

The nurse is assessing a client's blood pressure and is having difficulty hearing Korotkoff sounds. What is the most appropriate nursing intervention? -Wait a few minutes and then try to assess the BP. -Ask the client to open and close the fist after cuff inflation. -Contact the primary care provider for further instruction. -Ask the client to stand while assessing the BP.

airborne

The nurse is caring for a client with tuberculosis. Which precautions will the nurse select for this client? -contact -droplet -standard -airborne

auscultation of short, high-pitched popping sounds during inspiration

The nurse is assessing a client's thorax and lungs. Which finding would indicate the need for further assessment? -percussion of loud, hollow sounds over the lateral lung fields -auscultation of short, high-pitched popping sounds during inspiration -an anteroposterior to lateral ratio of 1:2 -palpation of tactile fremitus over the posterior thorax

Immediately assist the client back to bed

The nurse is assessing a female client for orthostatic hypotension. As the nurse assists the client to a standing position, the client states, "I'm feeling really dizzy." What should the nurse do next? -Apply the blood pressure cuff to check the blood pressure. -Immediately assist the client back to bed. -Inquire if the client would like to sit down. -Quickly ask the client to explain what she means by "dizzy."

Supplement the client's information by speaking with family or friends

The nurse is assessing a male client with a diagnosis of vascular dementia. As a result of his cognitive deficit, the client is unable to provide many of the data that are required on the hospital's nursing admission history document. How should the nurse best proceed with this assessment? -Limit the assessment to objective data. -Obtain the client's records from admissions to other institutions. -Perform the assessment in several short episodes rather than at one sitting. -Supplement the client's information by speaking with family or friends.

Listen for heart sounds

The nurse is assessing the apical pulse of a client using auscultation. What action would the nurse perform after placing the diaphragm over the apex of the heart? -Palpate the space between the fifth and sixth ribs. -Count each "lub-dub" as two beats. -Count the heartbeat for 2 minutes. -Listen for heart sounds.

The tympanic membrane is translucent, shiny, and gray

The nurse is assessing the ear canal and tympanic membrane of a client using an otoscope. Which finding would the nurse document as normal? -The tympanic membrane is reddish. -The ear canal is rough and pinkish. -The tympanic membrane is translucent, shiny, and gray. -The ear canal is smooth and white.

2+ pitting edema

The nurse is assessing the legs of a client and notes fairly normal contour with a 4 mm indentation when pressing on the shin and calf of each leg. How should the nurse interpret these findings? -Brawny edema -1+ pitting edema -2+ pitting edema -trace edema

"Put your arm in this sleeve."

The nurse is assisting an older adult client with dementia in getting dressed after morning care. Which statement would be most beneficial to the client? -"Don't put on your shoes yet." -"Put on your shirt." -"Put your arm in this sleeve." -"Put your pants on and zip the zipper."

Quality

The nurse is auscultating a client's abdomen and is able to hear gurgling sounds. When documenting this finding, which characteristic of sound is noted? -Texture -Quality -Turgor -Rhythm

Ask the client to cough and auscultate the anterior chest again

The nurse is auscultating the anterior chest of a client and hears gurgles. What is the nurse's appropriate action? -Document the findings. -Ask the client if they have any difficulty breathing. -Ask the client to cough and auscultate the anterior chest again. -Notify the healthcare provider.

142 beats per minute

The nurse is calculating the maximum heart rate for a 70-year-old client. Which accurately reflects the client's maximum heart rate? -200 beats per minute -175 beats per minute -142 beats per minute -125 beats per minute

immobilizer

The nurse is caring for a 19-year-old client who sustained a knee sprain while playing college sports. Which type of splint does the nurse anticipate will be ordered to support the knee while it heals? -molded -traction -immobilizer -inflatable

Apply a non-particulate (N-95) respirator when entering the room

The nurse is caring for a client admitted with tuberculosis (TB). What would be the best action by the nurse? -Wear a mask with face shield during invasive procedures. -Apply a non-particulate (N-95) respirator when entering the room. -Have the client wear a mask during care. -Wear a protective gown and gloves with any direct contact.

"I will brush once every day and floss every other day."

The nurse is caring for a client that has multiple dental caries and wants to understand the cause. Which statement by the client indicates that more teaching is warranted? -"I will get a dental check up every 6 months." -"I will brush once every day and floss every other day." -"I will rinse my mouth with water, if I am unable to brush." -"I will not chew ice cubes or crushed ice."

noncommunicable disease

The nurse is caring for a client who became very ill after ingesting seafood. How will the nurse document this condition? -communicable disease -infectious disease -noncommunicable disease -contagious disease

healthcare associated infection (HCAI)

The nurse is caring for a client who developed a urinary tract infection while hospitalized. How will the nurse document this condition? -contagious disease -healthcare associated infection (HCAI) -community acquired infection -infectious disease

bivalving of the cast

The nurse is caring for a client who had a cast applied yesterday for a tibia fracture. When the client returns today with swelling in that extremity, for which procedure does the nurse prepare? -surgical correction -application of petals to the cast -bivalving of the cast -application of a functional brace

Utilize prn doses of pain control medication

The nurse is caring for a client who has a long leg cast applied after surgery yesterday. The client is using a patient-controlled analgesia (PCA) device appropriately. At 1700, the client reports pain of "8" on a 1-10 scale. At 1900, pain has decreased to a "7". At 2100, the client reports pain of a "6". Which nursing intervention is appropriate? -Contact the family to sit with the client. -Activate the Rapid Response Team. -Utilize prn doses of pain control medication. -Notify the health care provider.

Launder gowns, linens, and towels separate from other clients items

The nurse is caring for a client who has been diagnosed with pediculosis. What intervention will the nurse provide? -Launder gowns, linens, and towels separate from other clients items. -There would be no intervention, as this chronic skin disorder is noninfectious. -Lesions should be squeezed gently to release pus. -Apply cortisone to this reddened rash.

Remove fresh fruit from the room

The nurse is caring for a client who has been placed in strict isolation. Which nursing action is appropriate? -No special precautions are required. -Remove fresh fruit from the room. -Deliver flowers and balloons to the room. -Allow many family members to visit at once.

skin

The nurse is caring for a client who has been placed into Buck's traction. Which type of traction intervention does the nurse document that this is? -skeletal -Russell's -skin -manual

administration of an antipsychotic agent to alter the client's behavior

The nurse is caring for a client who has been repetitively pulled at IV lines and the urinary catheter. After other methods of diverting the client's behaviors fail, the healthcare provider orders chemical restraints. Which treatment does the nurse anticipate? -articulating the reason for use of a physical restrictive device to the client's spouse -administration of an antipsychotic agent to alter the client's behavior -application of devices that reduce the client's ability to move arms -asking the unlicensed assistive personnel (UAP) to sit with the client

The client should expect to feel that the cast is hot and heavy

The nurse is caring for a client who has had a shoulder spica made of plaster of Paris applied. Which teaching will the nurse provide to the client? -The material will flex and bend with client movement. -Showering is important when wearing this type of cast. -The client should expect to feel that the cast is hot and heavy. -This device will be worn for 24-48 hours.

droplet

The nurse is caring for a client who requires frequent airway suctioning. Which precautions will the nurse select for the client? -respiratory -contact -droplet -airborne

"Consider the Alzheimer's Association 'Safe Return' program."

The nurse is caring for a client with Alzheimer's disease. A family member states, "I am afraid I will go to bed one night, and the next morning my loved one will be missing from wandering off." What is the appropriate nursing response? -"Clients with Alzheimer's disease often wander." -"I know, my parent has Alzheimer's disease and I worry about that too." -"Adjust sleeping schedules so that you can monitor your loved one as they sleep." -"Consider the Alzheimer's Association 'Safe Return' program."

The client's heart rate is greater than 90 bpm

The nurse is caring for a client with a stage IV leg ulcer. The nurse is closely monitoring the client for sepsis. What would indicate that sepsis has occurred and that the physician should be notified immediately? -The client's heart rate is greater than 90 bpm. -The client feels restless and hungry. -The client exhibits an increased urinary output. -The client's respiratory rate is less than 20 breaths/min.

contact

The nurse is caring for a client with acute viral conjunctivitis. Which precautions will the nurse begin? -contact -none -airborne -droplet

contact

The nurse is caring for a client with acute viral conjunctivitis. Which precautions will the nurse begin? -none -droplet -contact -airborne

Flush them down the toilet in the client's room

The nurse is caring for a client with an active upper respiratory infection. How will the nurse dispose of the client's unconsumed beverages and used paper tissues? -Put them in the waste can. -Flush them down the toilet in the client's room. -Double bag items for disposal. -Place them into the hazardous waste container.

molded

The nurse is caring for a client with carpal tunnel syndrome. Which type of splint does the nurse anticipate will be ordered? -immobilizer -traction -inflatable -molded

Contact a podiatrist to care for toenails

The nurse is caring for a client with diabetes who has thick toenails. What is the appropriate nursing intervention? -Clean under toenails with a wooden orange stick. -Use a handheld electric rotary file to reduce length of toenails. -Contact a podiatrist to care for toenails. -Clip the toenails with large clippers.

Sims'

The nurse is caring for a client with rectal bleeding. The nurse will place the client into which position to facilitate rectal examination? -Fowler's -Sims' -prone -supine

"Let's talk about how mobility may help you be more independent."

The nurse is caring for an older adult client who has difficulty walking. The client states, "It doesn't matter if I walk any more or not. I'm not much use to anyone." What is the appropriate nursing response? -"If you will try a walker you will be able to walk with more stability." -"Let's talk about how mobility may help you be more independent." -"Did someone tell you that you aren't of use to anyone?" -"I'm sure no one feels that way about you."

walker

The nurse is caring for an older adult client who is recovering from hip surgery. Which assistive device will the nurse use to facilitate client ambulation? -crutches -walker -cane -gait belt

The client is covered with a couple of thick blankets

The nurse is checking the client's temperature. The client feels warm to touch. However, the client's temperature is 98.8°F (37.1°C). Which statement could explain this? -The thermometer is broken. -A rectal thermometer must be used. -The client is showing initial signs of infection. -The client is covered with a couple of thick blankets.

hot water heater thermostat set at 130 degrees F (54.4 degrees C)

The nurse is conducting a home care visit for a new mother who delivered a baby 3 days ago. Which finding within the home requires immediate nursing intervention? -electrical outlets have covers over them -hot water heater thermostat set at 130 degrees F (54.4 degrees C) -infant's sleepwear is made from flame-resistant fabrics -one fire extinguisher is noted in the kitchen

Incident report

The nurse is documenting a variance that has occurred during the shift. This report will be used for quality improvement to identify high-risk patterns and, potentially, to initiate in-service programs. This is an example of which type of report? -Incident report -Transfer report -Telemedicine report -Nurse's shift report

handwashing

The nurse is getting ready to change the client's wound dressing. Which step best supports infection control? -handwashing -clean environment -sterile gloves -sterile gauze

"Make sure your family has all of their childhood immunizations."

The nurse is giving an educational talk to a local parent-teacher association. A parent asks how she could help her family avoid community-acquired infections. What would be the nurse's best response to help prevent and control community-acquired infections? -"Encourage your family to stop smoking." -"Make sure your family has all of their childhood immunizations." -"Make sure your family has regular checkups." -"Encourage your family to adopt a healthy diet and exercise regimen."

Working phase

The nurse is interviewing a client to obtain a nursing history. What phase of the interview process involves the nurse gathering all the information needed to form the subjective database? -Termination phase -Introductory phase -Working phase -Preparatory phase

asks the client to describe symptoms

The nurse is interviewing a client who is reporting chills, fever, malaise, and cough. During the working phase of the client interview, the nurse: -introduces self to client. -arranges for a private location. -summarizes the key points of the interview. -asks the client to describe symptoms.

"Can you tell me what you mean when you say 'fuzzy'?"

The nurse is interviewing a client who was admitted to the acute care facility. During the interview, the client states, "Sometimes I get a bit fuzzy after I take my medicine." Which response by the nurse would be most appropriate? -"What medications are you taking currently?" -"That's not unusual. I've heard several clients tell me the same thing." -"Are you experiencing lightheadedness with the medication?" -"Can you tell me what you mean when you say 'fuzzy'?"

Assess the client for dehydration

The nurse is palpating the skin of a 30-year old client and documents that when picked up in a fold, the skin fold slowly returns to normal. What would be the next action of the nurse based on this finding? -Report the finding as a positive sign for cystic fibrosis. -Assess the client for cardiovascular disorders. -Assess the client for dehydration. -Document a normal skin finding on the client chart.

a pronounced lateral curvature of the spine

The nurse is performing a physical assessment of an older adult female client. The nurse documents scoliosis as part of the spinal assessment. What is scoliosis? -a pronounced lateral curvature of the spine -an exaggerated lumbar curve of the spine -a gentle concave and convex curve of the spine -an increased curve in the thoracic area

coordinated movement of both eyes

The nurse is performing the positions test on a client following a head injury. Which assessment would the nurse interpret as a normal finding? -convergence of the eyes -nystagmus when looking in an upward position -coordinated movement of both eyes -limited movement in one eye when moving from superior to inferior position

shortness of breath after walking up five stairs

The nurse is planning care for a client with a nursing diagnosis of Activity Intolerance. What assessment finding would cause the nurse the most concern? -a change in pulse from 80 to 84 after walking up 20 stairs -shortness of breath after walking up five stairs -joint stiffness after sitting for an hour -walking with a slow and uncoordinated movement

The cast will be removed with an electric cast cutter

The nurse is preparing a client for cast removal. Which teaching will the nurse provide to the client? -Lotion cannot be used after casting. -The cast will be removed with an electric cast cutter. -Skin will be cleansed with alcohol to remove cast residual. -The muscle that was within the cast will be strong.

deep in the posterior sublingual pocket

The nurse is preparing to assess a client's oral temperature. The nurse should plan to place the thermometer probe in which area of the client's mouth? -along either upper gum line adjacent to an incisor -deep in the posterior sublingual pocket -in the inferior buccal space on either side of the tongue -superior to the tongue with the tip touching the hard palate

Provide privacy for the client

The nurse is preparing to assess a rectal temperature on an adult client. What is the appropriate nursing intervention? -Position the client on the stomach. -Provide privacy for the client. -Insert the thermometer 0.5 in (1.25 cm) into the rectum. -Maintain probe position in rectum for 2 minutes.

Lightly compress the client's radial artery using the first, second, and third fingers

The nurse is preparing to assess the peripheral pulse of an adult client. Which action is correct? -Compress the radial artery until no pulsation is felt, then gently remove the fingertips until the pulsation returns. -Lightly compress the client's radial artery using the first, second, and third fingers. -Encircle the client's antecubital fossa with both hands and lightly compress the brachial artery with the first fingers of both hands. -Grasp the client's inner wrist with the nondominant thumb positioned over the radial artery.

Lightly compress the client's radial artery using the first, second, and third fingers

The nurse is preparing to assess the peripheral pulse of an adult client. Which action is correct? -Encircle the client's antecubital fossa with both hands and lightly compress the brachial artery with the first fingers of both hands. -Lightly compress the client's radial artery using the first, second, and third fingers. -Grasp the client's inner wrist with the nondominant thumb positioned over the radial artery. -Compress the radial artery until no pulsation is felt, then gently remove the fingertips until the pulsation returns.

Listen to at least one full respiration in each location

The nurse is preparing to auscultate the lung sounds of a client with asthma. How should the nurse complete the assessment? -Listen as the client inhales and then move the next site during exhalation. -Auscultate lung sounds over the client's clothing or hospital gown. -Listen to at least one full respiration in each location. -Instruct the client to rapidly breathe in and out the nurse listens.

Organize the assessment so the client does not change positions too often

The nurse is preparing to perform a head-to-toe physical assessment. What approach will the nurse use? -Perform the examination from the left side of the bed. -Organize the assessment so the client does not change positions too often. -Begin by examining the thorax and abdomen. -Examine the structures in each system separately.

Wear PPE, Keep client's environment clean, Practice hand hygiene

The nurse is providing an inservice educational program for the interprofessional healthcare team about infection control precautions. What teaching will the nurse include? (Select all that apply.) -Wear PPE. -Keep client's environment clean. -Use standard precautions only for patients with infection. -Practice hand hygiene. -Use equipment repetitively on clients with similar conditions.

parasite

The nurse is providing care to a client with Lyme disease. The nurse identifies the cause of this infection as: -parasite. -bacteria. -virus. -fungus.

Dry the cleaned areas and apply an emollient as indicated

The nurse is providing perineal care for clients in a hospital setting. What is an appropriate nursing action when providing this type of care? -Do not retract the foreskin in an uncircumcised male. -Dry the cleaned areas and apply an emollient as indicated. -Powder the area to prevent the growth of bacteria. -Always proceed from the most contaminated area to the least contaminated area.

Suggest to the nurse manager that an in-service on abbreviation use would be helpful

The nurse is reading another nurse's notes that were recorded in the electronic health record (EHR) during the previous shift. What is the appropriate nursing action when numerous unapproved abbreviations are noticed in the previous nurse's notes? -Suggest to the nurse manager that an in-service on abbreviation use would be helpful. -Ask another nurse to fix the abbreviations. -Correct the abbreviations in the EHR. -Contact the facility's information technology department to delete abbreviations.

into a private room

The nurse is receiving a confused client with a draining wound onto the medical-surgical unit. Which room assignment will the nurse make? -with a client with pneumonia -with a client with a myocardial infarction -with another client with a draining wound -into a private room

"Once I start feeling better, I should stop taking the antibiotic."

The nurse is reviewing discharge instructions for a client who was prescribed an antibiotic. Which statement by the client would require further teaching? -"I should avoid sharing my antibiotic with my spouse." -"If I develop a rash, I will contact my healthcare provider." -"Once I start feeling better, I should stop taking the antibiotic." -"I have a bacterial infection that requires an antibiotic."

an inference

The nurse is reviewing information about a client and notes the following documentation: "Client is confused." The nurse recognizes this information is an example of: -primary data. -an inference. -a data cue. -subjective data.

an older adult client with a history of heart failure

The nurse is reviewing the plan of care for assigned clients. Which client has the highest risk for developing an infection? -a middle-aged adult who takes prescribed medication to control blood pressure -an adolescent who has a right radial fracture -a school-age child who is current with immunizations -an older adult client with a history of heart failure

A= Assessment

The nurse is speaking to the physician regarding the client's frequent diarrhea episode since starting IV antibiotics. The nurse states "I am concerned that Mr. Clark has developed Clostridium Difficile infection". Which part of the SBAR communication will this statement fall into? -R= Recommendation -S= Situation -A= Assessment -B= Background

Termination phase

The nurse is summarizing the key points of the interview. This nursing activity occurs during which phase? -Preparatory phase -Introductory phase -Working phase -Termination phase

Store the hearing aid in cool environment

The nurse is teaching a client about hearing aid care. Which teaching is appropriate? -Use sandpaper to clean off hardened cerumen residue. -Do not change batteries by yourself. -Clean the hearing aid with a bleach. -Store the hearing aid in cool environment.

moves legs away from the midline, then toward the midline

The nurse is teaching a client about moving joints into a position of abduction and adduction. Which client action reflects that teaching has been effective? -turns arms downward, and then upward -moves legs away from the midline, then toward the midline -turns sole of foot toward the midline, then away from the midline -tilts chin down to touch chest, then stretches head back comfortably far

"I will tighten the belt when I go to sleep."

The nurse is teaching a client who had a below-the-knee amputation about a temporary prosthetic limb. Which client statement requires further nursing teaching? -"If I have an incisional infection, I will not wear the limb." -"I will tighten the belt when I go to sleep." -"This device will help to control swelling of the stump." -"This is a lightweight artificial limb."

Hold the walker at the padded handgrips

The nurse is teaching a client who is recovering from surgery about using a walker. Which teaching will the nurse include? -Stand just behind the walker. -Hold the walker at the padded handgrips. -Pick up the walker and advance it 10 to 12 inches. -Move the walker while taking a step forward.

The new nurse touches 1.5 in (4 cm) from the outer edges

The nurse is teaching a new nurse about preparing a sterile field. Which action made by the new nurse would indicate further teaching is required? -The sterile field is set up at waist level. -The new nurse touches 1.5 in (4 cm) from the outer edges. -The top flap of the package is opened away from the new nurse's body. -Direct visualization of the sterile field is maintained.

Wash hands thoroughly after removing gloves with a pH balanced soap

The nurse is teaching a nursing student about proper latex glove use. Which teaching will the nurse include? -Use powdered gloves. -Wash hands thoroughly after removing gloves with a pH balanced soap. -Use hand cream or lotion after removing gloves to preserve skin integrity. -Snap the gloves when applying them to ensure proper fit.

"I will rescue clients from harm before doing anything else."

The nurse is teaching an unlicensed assistive personnel (UAP) about fire safety. Which UAP statement demonstrates that teaching has been effective? -"I will rescue clients from harm before doing anything else." -"I know that nurses are the only ones who can extinguish a fire." -"I will leave all doors open after rescuing patients." -"I will sound the alarm before I start moving a patient from a room."

trapeze bar

The nurse is working to increase functional ability with a client. Which assistive technique should be included in the plan of care? -trapeze bar -pull sheets -log rolling -trochanter rolls

The Joint Commission (TJC)

The nurse know that this organization is an accrediting body for healthcare organizations. -National Institute for Occupational Safety and Health (NIOSH) -The Joint Commission (TJC) -American Nurses Association (ANA) -National League for Nursing (NLN)

closing of the aortic and pulmonic valves

The nurse knows that the heart sound S2 represents -opening of mitral and tricuspid valves. -closing of the mitral and tricuspid valves. -opening of the aortic and pulmonic valves. -closing of the aortic and pulmonic valves.

discussing a recent event when a newly-admitted client went into cardiac and respiratory

The nurse manager on a medical unit has scheduled a debriefing session. Which of the following is the most likely subject of this meeting? -introducing a new group of student nurses who will soon begin a rotation on the unit -discussing a recent event when a newly-admitted client went into cardiac and respiratory arrest -describing the correct use of a new type of venous access device that will soon be introduced -eliciting feedback about a proposed change in the scheduling procedures on the unit

The client has fallen before.

The nurse needs to plan the interventions necessary to reduce fall risks for the older adult clients at her facility. Which is the strongest indicator that a client is at risk for falls? -The client is forgetful. -The client is on beta blockers and antihypertensive medication. -The client uses a rolling walker. -The client has fallen before.

removes gloves and walks out of the room

The nurse observes an unlicensed assistive personal (UAP) collecting a urine specimen from a client with Staphylococcus aureus infection. Which action by the UAP would require the nurse to intervene? -removes gloves and walks out of the room -applies a mask with face shield -performs hand hygiene before donning gloves -asks the client to state name and date of birth

Orthopnea

The nurse places a client experiencing labored breathing in an upright position. The nurse notes that the client is able to breathe more easily in this upright position and documents this condition on the chart as: -Apnea -Bradypnea -Orthopnea -Tachypnea

Orthopnea

The nurse places a client experiencing labored breathing in an upright position. The nurse notes that the client is able to breathe more easily in this upright position and documents this condition on the chart as: -Apnea -Tachypnea -Orthopnea -Bradypnea

National Institute for Occupational Safety and Health (NIOSH)

The nurse recognizes that which organization requires that employers comply with ergonomic recommendations? -National Institute for Occupational Safety and Health (NIOSH) -The Joint Commission (TJC) -American Nurses Association (ANA) -National League for Nursing (NLN)

"Pieces of moleskin that cover rough edges of the cast."

The nurse tells a newly casted client that "petals" will be applied. What is the appropriate nursing response when the client asks, "What are petals?" -"A type of plaster that reinforces the strength of the cast." -"Porous material from which the cast is made." -"Pieces of moleskin that cover rough edges of the cast." -"Large bandages that surround the cast to keep it in place."

stethoscope that remains in the client's room

The nurse who is caring for a client in contact isolation is preparing to conduct an assessment. How will the nurse listen to the client's heart? -stethoscope that has been purchased by the client -stethoscope that remains in the client's room -stethoscope that hangs outside the client's room -stethoscope belonging to the nurse

Palpation

The nurse will obtain the greatest amount of information about the thyroid gland by using which technique of assessment? -Inspection -Percussion -Palpation -Auscultation

-Walking -Swimming -Bicycling

The nurse would like the client to perform some exercises that use muscle shortening and active movement. The nurse tells the client it will help build bone and improve cardiac and respiratory functioning. Which exercises should the nurse encourage the client to do? Select all that apply. -Walking -Yoga exercises -Swimming -Bicycling -Lifting weights

Toes are dry and cool to touch

The urgent care nurse is assessing a client who had a plaster cast applied to the left ankle 4 hours ago. The client has come back to the urgent care with concerns. Which finding requires immediate nursing intervention? -Shape of cast has changed slightly. -Toes are dry and cool to touch. -Capillary refill is 3 seconds. -Client reports pain of "4" on 1-10 scale.

Problem-oriented recording emphasizes goal-directed care to promote the recording of pertinent data that will facilitate communication among health care providers.

The nurses at a health care facility were informed of the change to organize the clients' records into problem-oriented records. Which explanation could assist the nurses in determining the advantage of using problem-oriented records? -Problem-oriented recording is difficult to demonstrate a unified approach for resolving the clients' problem among caregivers. -'Problem-oriented recording gives the clients the right to withhold the release of their information to anyone. -Problem-oriented recording emphasizes goal-directed care to promote the recording of pertinent data that will facilitate communication among health care providers. -Problem-oriented recording has numerous locations for information where each member of the multidisciplinary team makes entry about their own specific activities in relation to the client's care.

The nurse should show her name badge to the client so he can identify the nurse

The nursing instructor is teaching the students how to do an interview on a client. Which statement made by a student indicates a need for further instruction? -The nurse should verify the client's name. -The nurse should ask the client what name he would like to be called. -The nurse should show her name badge to the client so he can identify the nurse. -The nurse should introduce herself and give name and position. -The nurse should sit on eye level with the client.

care bundle

The nursing leaders on an intensive care unit have introduced a new protocol for the care of clients who are receiving parenteral nutrition. All of the major nursing assessments and interventions relevant to parenteral nutrition have been combined into one list. This unit has introduced a: -standard of care. -care bundle. -nursing care plan. -nursing policy.

asking the client whether the client has cultural preferences

The nursing student demonstrates accurate application of the assessment phase of the nursing process by performing which action? -determining if the client's goals for wellness have been met -developing a plan to manage the client's health problems -asking the client whether the client has cultural preferences -coming up with the nursing diagnosis based on potential health risk

blood pressure 140/90 mm Hg

The occupational nurse is assessing an employee's vital signs at rest. Which finding requires nursing intervention? -pulse rate 88 beats per minute -temperature 98.8ºF -respirations 18 per minute -blood pressure 140/90 mm Hg

"Both of your feet should rest on the floor."

The occupational nurse is teaching an administrative assistant about proper posture when sitting. Which teaching will the nurse include? -"Cross your legs alternately throughout the day." -"Keep your knees bent, with the back of the knees against your chair." -"Upper and lower thighs are your base of support." -"Both of your feet should rest on the floor."

just before the 6 a.m. dose

The physician orders a serum trough drug level for a client who is receiving antibiotic therapy. The client is receiving the drug every 6 hours: at midnight, 6 a.m., noon and 6 p.m. The nurse anticipates that the specimen would be obtained: -at 6 p.m. -immediately after the noon dose. -just before the 6 a.m. dose. -at 7 a.m.

"Is your child breathing at this time?"

The poison control nurse receives a call from the parent of a 2-year-old child. The parent states, "I just took a quick shower, and when I finished, I walked into the kitchen and found my child with an open bottle of household cleaner." What is the poison control nurse's appropriate response? -"You should not have left your child alone while you showered." -"Did you leave the household chemical in reach of your child?" -"Is your child breathing at this time?" -"Induce vomiting and call 9-1-1 right away."

Assess client's pain level and manage pain accordingly

The postoperative client refuses to do deep breathing, and he refuses to turn while in bed. He informs the nurse that it hurts for him to do both of these things. Which intervention should the nurse perform first? -Inform the physician of the client's noncompliance -Assess client's pain level and manage pain accordingly. -Inform the client that these exercises must be done at regular intervals. -Educate the client of the importance of infection prevention.

ecchymosis

The presence of purple patches on the skin, due to trauma to soft tissue, is documented as -Pallor -Ecchymosis -Flushed -Erythema

cheyne-strokes

abnormal characterized by progressively deeper and sometimes faster breathing followed by a gradual decrease that results in a temporary stop in breath called an apnea

use of elevators

There is a fire in the neurology unit of a health care facility. What would be most appropriate to avoid in this situation? -use of stretchers -use of elevators -clamping of clients' suction tubes -closing of doors and windows

neutrophils

These types of white blood cells are Phagocytes. They ingest and break down foreign particles, particularly bacteria and parasites. They are also an important link in generating fever to combat the proliferation of microorganisms.

Tinea capitis

This is rounded patchy hair loss on the scalp, leaving broken-off hairs, pustules, and scales on the skin caused by a fungal infection

convalescent

This period describes when body systems return to normal and appetite and energy return; antibodies begin to appear in the person's blood

incubation

This period is the time between the pathogen's entrance and appearance of symptoms.

prodromal

This phase is characterized by nonspecific symptoms such as nausea, fever, weakness, aches, or pains.

Dangle feet before moving from a reclining position.

To avoid postural hypotension, which teaching will the nurse provide to the client? -Eat something with protein before getting up in the morning. -Rise from lying down quickly. -Apply antiembolism stockings before rising. -Dangle feet before moving from a reclining position.

Coughing

What would be considered a mechanical defense mechanism? -Coughing -Cast -Sunscreen -Clothing

air

Tuberculosis is transmitted via the?

not all of the heartbeats are reaching the periphery

Two nurses collaborate in assessing an apical-radial pulse on a client. The pulse deficit is 16 beats/min. What does this indicate? -This is a normal finding and should be ignored. -Not all of the heartbeats are reaching the periphery. -The client's arteries are very compliant. -The radial pulse is more rapid than the apical pulse.

Monitor white blood cell count

Upon cast assessment of a client with an open tibia fracture, the nurse notices yellowish drainage coming from within the cast. Which nursing intervention is appropriate? -Pack gauze around the top and bottom of the cast to absorb drainage. -Withhold opioid medication prescribed for pain. -Monitor white blood cell count. -Place an ACE bandage around the cast to provide stability.

Provide the client with an electric shaver

Upon review of the client's orders, the nurse notes that the client was recently started on an anticoagulant agents. What is an appropriate consideration when assisting the client with morning hygiene? -Provide the client with a firm bristled toothbrush. -Provide the client with an electric shaver. -Avoid massaging the client's back with lotion. -Do not allow the client to shower.

Mummy restraint

Use of which restraint requires the nurse to stay with the client until the restraint is discontinued? -Mummy restraint -Extremity restraint -Vest restraint -Elbow restraint

Systolic pressure

Various sounds are heard when assessing a blood pressure. What does the first sound heard through the stethoscope represent? -Pulse pressure -Diastolic pressure -Systolic pressure -Auscultatory gap

Cardiac sinoatrial (SA) node

What anatomic site regulates the pulse rate and force? -Peripheral chemoreceptors -Thermoregulatory center -Cardiac sinoatrial (SA) node -Cardiac atria and valves

Loudest at the apex

What area of the chest (precordium) is S1 heard the loudest?

Loudest at the base

What area of the chest (precordium) is S2 heard the loudest?

oral: 37°C

What is an average normal temperature in Celsius for a healthy adult? -tympanic: 34.4°C -rectal: 36.5°C -oral: 37°C -axillary: 37.5°C

to plan appropriate nursing care

What is the primary purpose of validation as a part of assessment?

Gown, gloves, and mask with eye shield

What type of PPE would be appropriate for assisting with respiratory care of a pediatric patient with pertussis (whooping cough)? -Gown and gloves -Gown, gloves, and mask -Gown, gloves, and mask with eye shield -Gown, gloves, and respirator

auscultatory gap

absence of Korotkoff sounds between phases I & II

Respirations have both autonomic and voluntary control

When assessing a client's vital signs, a nursing student has explained to the client each of her next actions prior to assessing the client's temperature, pulse, and blood pressure. However, the nurse has not announced her intention to assess the client's respiratory rate prior to measuring it. What is a plausible rationale for the nurse's decision? -Temperature, pulse, and blood pressure are more volatile than respiratory rate. -Tachypnea is an expected finding among hospitalized individuals. -The nurse likely assessed the client's respiratory rate simultaneous to heart rate. -Respirations have both autonomic and voluntary control.

the weight of the body is distributed on the soles and heels

When assessing for proper body alignment of a standing client, what is a normal finding? -The abdominal muscles are held downward and the buttocks upward. -The chest is downward and displaced slightly backward. -The line of gravity is deviated slightly to the left -The weight of the body is distributed on the soles and heels.

the middle-age computer programmer

When assessing the physical activity of clients, the nurse would be most concerned about which client? -the Native American who hunts -the older adult client who goes to the mall 3 times a week -the young mother of a 2-year-old and 4-year-old -the middle-age computer programmer

-having the client wear a mask when outside the room -removing fresh flowers from the room -placing the client in a private room

When developing a plan of care for a client who has developed neutropenia secondary to chemotherapy, which of the following would the nurse most likely include? Select all that apply. -having the client wear a mask when outside the room -measuring temperatures rectally -removing fresh flowers from the room -placing the client in a private room -providing vigorous oral care

Fold soiled side to the inside and roll with inner surface exposed

When discontinuing use of a gown in the care of a client in droplet precautions, which method does the nurse use to dispose of this personal protective equipment (PPE)? -Fold soiled side to the outside and roll with inner surface exposed. -Fold soiled side to the outside and roll with outer surface exposed. -Fold soiled side to the inside and roll with inner surface exposed. -Fold soiled side to the inside and roll with outer surface exposed.

Toddler

When performing an assessment, the nurse should focus on the developmental stage for which client? -Adolescent -Middle-age adult -Toddler -Young adult

a thorough, mechanical cleaning

When providing oral care, what does the nurse recognize as the most important component of the oral care process? -application of moisturizing ointment to the lips -a thorough, mechanical cleaning -use of a mouthwash or breath freshener -selection of toothpaste

white blood cell count of 14,000/mL

When providing pin care for a client who has an external fixator, which assessment finding requires the nurse to intervene? -serosanguineous draining is noted -client uses patient-controlled analgesia (PCA) pump -crusted secretions are present around pins -white blood cell count of 14,000/mL

The systolic reading is below 100 and diastolic reading is below 60

Which client's blood pressure best describes the condition called hypotension? -The systolic reading is above 102 and diastolic reading is above 60. -The systolic reading is below 120 and the diastolic reading is below 80. -The systolic reading is below 100 and diastolic reading is below 60. -The systolic reading is above 110 and diastolic reading is above 80.

During ventricular relaxation, blood pressure is due to elastic recoil of the vessels.

Which describes diastolic blood pressure? -The pressure is highest when the ventricles of the heart eject blood into the aorta and pulmonary arteries. -During ventricular relaxation, blood pressure is due to elastic recoil of the vessels. -the flow of blood produced by contractions of the heart and the resistance to blood flow through the vessels -the blood pressure measured during ventricular contraction

asepsis

absence of disease producing microorganisms

Young children

Which age group is most vulnerable to toxic fumes or asphyxiation? -Young children -Young adults -Adolescents -Middle adults

Cane

Which ambulatory aid could a nurse suggest to assist a client who has weakness in one side of his body? -Forearm crutch -Walker -Axillary crutch -Cane

wearing a particulate respirator for all client care and interaction

Which care intervention should the nurse anticipate when providing care to a client admitted with a possible diagnosis of tuberculosis (TB)? -wearing a particulate respirator for all client care and interaction -placing the client in a regular, private room -wearing a face mask when entering and staying at a distance from the client -wearing protective eye wear for all client contact

a client recovering from a bone marrow transplant

Which client is most likely to require neutropenic precautions? -a client awaiting a liver transplant -a client diagnosed with tuberculosis -a client recovering from orthopedic surgery -a client recovering from a bone marrow transplant

an 81-year-old client who has been receiving multiple antibiotics for the treatment of sepsis

Which client presents the most significant risk factors for the development of Clostridium difficile infection? -A client with renal failure who receives hemodialysis three times weekly -an 81-year-old client who has been receiving multiple antibiotics for the treatment of sepsis -A 30-year-old client who has recently contracted human immunodeficiency virus (HIV) after engaging in high-risk sexual behavior -a 44-year-old client who is paralyzed and whose coccyx ulcer has required a skin graft

An older adult resident of an extended-care facility is being assessed by a nurse practitioner during the nurse's scheduled monthly visit.

Which client situation most likely warrants a time-lapse nursing assessment? -The nurse has responded to the call light of a hospital client who is reporting shortness of breath and chest pain. -A client is being admitted to a general medicine unit after spending several days in the intensive care unit. -A nurse is auscultating the lungs and measuring the oxygen saturation of a client who has pulmonary edema. -An older adult resident of an extended-care facility is being assessed by a nurse practitioner during the nurse's scheduled monthly visit.

an alert client after knee replacement surgery who is being assisted to ambulate

Which client would be an appropriate candidate to move by using a powered stand-assist device? -an alert client after knee replacement surgery who is being assisted to ambulate -a car accident victim with fractures in both legs who is being moved to another room -a comatose client who is being taken for x-rays -an obese client who has Alzheimer's disease and is being escorted to the shower room

Adjusting reimbursement based on measurement of processes, outcomes, and patient satisfaction

Which description of value-based purchasing is most accurate? -Ensuring "never events" never occur -Conducting a thorough investigation to analyzing the root cause of all errors -Adjusting reimbursement based on measurement of processes, outcomes, and patient satisfaction -Determining the actual cost of care to calculate reimbursement

Sweat

Which factors are identified as the body's defense against infection? -Sweat -Saliva -Interferon -Peristalsis -Acidic pH of the vagina

Collection, validation, communication of client data

Which group of terms best defines assessing in the nursing process? -Design a plan of care, implement nursing interventions -Nurse-focused, establishing nursing goals -Collection, validation, communication of client data -Problem-focused, time-lapsed, emergency-based

"Assessment data about the client should be collected continuously."

Which guideline should a nursing instructor provide to nursing students who are now responsible for assessing their clients? -"Assess your client at least hourly if the client's vital signs are unstable, and every 2 hours if the vital signs are stable." -"Assessment data should be collected prior to the physician rounding on the unit." -"Assessment data about the client should be collected continuously." -"Assess your client after receiving the nursing report and again before giving a report to the next shift of nurses."

use of one gown per person per shift

Which is not appropriate regarding the use of gowns as PPE? -Don a gown when splashing. -use of paper or cloth gowns -use of one gown per person per shift -use of a new gown each time the nurse enters the room

Metabolism

Which is the primary source of heat in the body? -Metabolism -Muscles -Blood circulation -Hormones

Nurse practitioner

Which level of health care provider may make the decision to apply physical restraints to a client? -LPN team leader -Senior personal care assistant -Nurse practitioner -RN nurse manager

airborne precautions droplet precautions contact precautions

Which of the following are names of the transmission-based precautions defined by the Centers for Disease Control (CDC)? Select all that apply. -airborne precautions -droplet precautions -contact precautions -respiratory precautions -microbial precautions -body fluid precautions

Temperature, turgor, moisture

Which of the following can a nurse assess by palpation? -Vision, hearing, cranial nerves -Tissue density, gait, reflexes -Temperature, turgor, moisture -Heart sounds, lung sounds, blood pressure

Natalia is a visiting nurse who has an appointment with Donald, an 85-year-old man with mobility issues. Natalia has worked with Donald in the past on the ways in which he can prevent falls. Today she wants to assess how he is doing with the fall prevention strategies they practiced before.

Which of the following is an example of a time-lapse reassessment? -Bob is a nurse in a long-term skilled nursing facility. Noreen is a new client. Bob wants to gather information from Noreen, which includes her health status and any problematic health patterns, and to get a baseline for Noreen's overall functioning. -Joan is a nurse who is just coming on to her shift. She has received client reports from the nurse leaving the floor. To start off her day, she goes into each of her client's rooms and performs a focused physical assessment based on each individual's diagnosis. -Daren is a nurse in a hospital who happens to walk by a room and notices a client down on the floor. Daren immediately assesses the client for airway, breathing, and circulation. Once the presence of these three is established, Daren calls for help and begins a quick neurological exam. -Natalia is a visiting nurse who has an appointment with Donald, an 85-year-old man with mobility issues. Natalia has worked with Donald in the past on the ways in which he can prevent falls. Today she wants to assess how he is doing with the fall prevention strategies they practiced before.

Nurses play an important role in keeping patients safe.

Which statement is correct about patient safety? -Physicians have sole responsibility for patient safety. -Safe patient care is the same as quality care. -High scores on HCAHPS surveys indicate patient safety. -Nurses play an important role in keeping patients safe.

They are errors that are due to system issues

Which statement is true about latent errors? -They are errors whose effects are not known for a long time. -Latent errors cause minimal effects to patients. -Latent errors are traceable to a particular individual. -They are errors that are due to system issues.

Sympathetic nervous system activation occurs in response to a variety of stimuli, including changes in intravascular volume

Which statement is true regarding the autonomic nervous system and its effect on the rate of a person's pulse? -Stimulation of the sympathetic nervous system results in a decrease in the pulse rate. -Sympathetic nervous system activation occurs in response to a variety of stimuli, including changes in intravascular volume. -Stimulation of the parasympathetic nervous system results in an increase in the pulse rate. -The sympathetic nervous system is the dominant activation during resting states.

Bring a narrow beam of light from the temple toward the eye, observing for direct and consensual pupillary constriction

Which technique should the nurse use to assess the pupillary light reflex on a client? -Use an ophthalmoscope to focus light on the sclera and observe for a reflection on each eye. -Ask the client to follow the penlight in six directions and observe for bilateral pupil constriction. -Have the client focus on a distant object, then ask the client to look at the penlight being held about 4 cm from the nose and observe for pupil constriction. -Bring a narrow beam of light from the temple toward the eye, observing for direct and consensual pupillary constriction

Palpate the pulse for 1 minute

Which technique should the nurse use when assessing the radial pulse of a client with a history of atrial fibrillation? -Palpate the pulse for 10 seconds and multiple by 6. -Palpate the pulse for 1 minute. -Palpate the pulse for 2 minutes. -Palpate the pulse for 15 seconds and multiply by 4.

Pyrexia

Which term indicates a potentially serious client condition? -Pyrexia -Eupnea -Pulse pressure -Afebrile

keeping medications in clearly labeled containers

Which topic should a public health nurse emphasize when educating older adults on reducing their risk of poisoning? -alternatives to chemical-based cleaning supplies -hidden sources of lead in the household -environment avoiding the use of alternative and complementary therapies -keeping medications in clearly labeled containers

Focused assessment

Which type of assessment would the nurse be expected to perform on the client who is 1 day postoperative following a cholecystectomy? -Time-lapse assessment -Initial assessment -Emergency assessment -Focused assessment

a cane with four prongs on the end (quad cane)

Which type of mobility aid would be most appropriate for a client who has poor balance? -a single-ended cane with a half-circle handle -axillary crutches -a single-ended cane with a straight handle -a cane with four prongs on the end (quad cane)

unsteady gait

While assessing a client admitted with a transmissible spongiform encephalopathy, what finding might the nurse observe? -unsteady gait -difficulty breathing -distended abdomen -redden, circular rash

Moisture in air passages

While assessing breath sounds, a nurse hears crackles. What causes these abnormal sounds? -A narrowing of the upper airway -Air in the lungs -Moisture in air passages -Narrowed small air passages

a crack in the skin, especially in or near a mucous membrane

While assessing the characteristics of the skin of the client, the nurse observes a mouth slit at the aperture of the mouth. The nurse documents this finding as a fissure. What is a fissure? -an area of the skin that has been rubbed away by friction -a crack in the skin, especially in or near a mucous membrane -a mark left on the skin by the healing of a wound or lesion -an open, crater-like area on the skin

Contractures

While performing passive range-of-motion exercises on the lower extremities of a client with a spinal cord injury, the nurse assesses permanent flexion of the muscles. What term will the nurse use to document this finding related to the muscles? -Ankylosis -Tonus -Atrophy -Contractures

Discard the sterile field to start over

While setting up a sterile field, a small of amount of water splashes onto the sterile drape. Which action by the nurse would be most appropriate? -Quickly dab up the water with a sterile dressing. -Discard the sterile field to start over. -Place a nonabsorbent pad over the damp area. -Avoid using that area because it is unsterile.

to promote health and prevent illness

Why is it important for the nurse to teach and role model proper body mechanics? -to prevent unnecessary insurance claims -to demonstrate knowledge and skills -to ensure knowledgeable client care -to promote health and prevent illness

The nationally recognized catheter-associated urinary tract infection prevention bundle

You have been asked to participate in a committee writing a policy for the care of a patient with an indwelling catheter. What is most important to include? -Information retrieved from a website directed toward the lay public -The nationally recognized catheter-associated urinary tract infection prevention bundle -The policy from a well-respected nearby hospital -Information from a nursing textbook that is more than 10 years old

Conducting root cause analysis

Your nurse manager informs you he is using the tool "Asking Why 5 Times" to investigate medication error in which you were involved. What was the nurse manager doing? -Conducting root cause analysis -Applying concepts of just culture -Assessing outcomes for CMS -Using bundles of care

Bradypnea

______ is a decrease in respiratory rate

Tachypnea

________ is an increased respiratory rate

Apnea

________ refers to periods during which there is no breathing

elderly

a 4+ pulse is considered to be normal in what type of patients?

susceptible host

a person whose own bodies defense mechanisms cannot withstand the invasion of the pathogen when it is exposed to a pathogen

percussion

aided to detecting masses, fluid or air in the peritoneum?

basal metabolic rate (BMR)

amount of energy the body uses or the amount of heat produced at rest. can be increased by activity

respiration rate 25, heart rate 114, crackles on auscultation

an older adult has limited mobility as a result of a hip fracture. which of the following objective date support possible pulmonary problems related to impaired mobility? select all that apply -Blood pressure 128/84 -respiration rate 26 -heart rate 114 -crackles on auscultation -pain 3 on a scale of 0-10

Crackles

are intermittent, high-pitched, popping sounds, which are heard in distant areas of the lungs during inspiration. -Rubs -Crackles -Wheezes -Gurgles

Gurgles

are low-pitched, continuous, bubbling adventitious sounds, which are prominent during expiration, and are heard in larger airways -Rubs -Crackles -Wheezes -Gurgles

Wheezes

are whistling or squeaking sounds caused by air moving through a narrow passage, which can be heard throughout the chest during expiration or inspiration. -Rubs -Crackles -Wheezes -Gurgles

murmurs

these are vibrating wishing blowing sounds that can be heard through both systole and diastole

extended-spectrum beta-lactamases (ESBLs)

enzymes that give bacteria immunity to both penicillin and cephalasprin antibiotics

systole

contraction of the heart muscles, especially the ventricles

athetosis

this is movement characterized by slow irregular twisting motions

tremor

this is rhythmic receptive movement occurring at rest or when movement is initiated

77F or 113F

death can occur in a patient if the core temperature reaches this?

kussmual

deep and laboring breathing patter often associated with sever metabolic acidosis, particularly diabetic ketoacidosis but also kidney failure Sever metabolic acidosis

800 cells/mm3

determines that the client is experiencing neutropenia based on which absolute neutrophil count? -1,100 cells/mm3 -1,500 cells/mm3 -800 cells/mm3 -1,800 cells/mm3

the occupational safety and health administration (OSHA)

develops standards and regulations to protect the safety and health of workers

gallops

extra heart sounds such as S3 and S4 are known as what?

The client's weak pulses may be indicative of cardiovascular disease

he nurse has palpated a client's radial pulses bilaterally and has documented the results of this assessment as "radial pulses 1+ bilaterally." How should this assessment finding be interpreted? -The client shows no signs of a circulatory health problem. -The client has increased radial pulses that may result from hypertension. -The client has normal peripheral pulses. -The client's weak pulses may be indicative of cardiovascular disease

2 hours

how often should immobilized patients be turned and reposition

peristalsis

hyperactive bowl sounds can be a cause of this? which is involuntary contractions and relaxation of the muscles of the intestine or another canal, creating wave like movements that push the content of the canal forward

3 minutes

if you can not hear bowl sounds in the 4 quadrants how long should you listen for before noting that no bowl sounds are present?

Eupnea

means a normal breathing pattern

Afebrile

means that the body temperature is not elevated

pathogens

microorganism that can cause harm to humans

parasites

multicellular organismi that live on other organisms without contributing anything to their host

prions

organisms that cause a rapidly progressing neurogenerative disease affecting both animals and humans that is untreatable and always fatal

5.5

output of blood by the heart per 1 minute averages about how many L/min

portal of entry

permits the organism to gain entrance to the host

infection disease

process resulting from infection that produces manifestations such as fever, leukocytosis, inflammation or tissue damage

centers for disease control (CDC)

publishes guideline on both standard and transmission based patients

pyrexia

raised body temp (fever)

the joint commission

sets requirements that healthcare agencies must meet to obtain accreditation

fungi

single cell organism that include mold and yeast

bacteria

single celled, independently living microorganism, some of which are capable of causing disease in humans

emesis

the action or process of vomiting

tidal volume

the amount of air moving in/out with each breath (6-8L/min)

stroke volume

the amount of blood pumped by the LV

skin

the bodies 1st line of defense against microorganism entering the body

hypothalamus

the body has a built in thermostat that helps regulate your bodies temperature. What is responsible for this job

"This antibiotic is the best choice since the causative organism is not known."

the client presents to the Emergency Department reporting fever, chills, and a productive cough. The chest x-ray shows an area of infiltrate, and the primary care provider prescribes a broad spectrum antibiotic. Which client teaching statement from the nurse is most appropriate regarding the prescribed drug? -"This antibiotic is the best choice since the causative organism is not known." -"This antibiotic causes fewer side effects than a narrow spectrum antibiotic." -"Pneumonia is usually caused by multiple organisms." -"Drug resistance can develop when the wrong antibiotic is used for pneumonia."

pulse pressure

the difference between systolic and diastolic

change to airborne precautions

the nurse is caring for a client with tuberculosis. The prior shift's nurse has placed the client in droplet precautions. Which is the appropriate nursing action? -Leave in droplet precautions. -Change to standard precautions. -Change to airborne precautions. -Change to contact precautions.

fomites

these are inanimate objects where pathogens find nourishment

Lactate

these levels help identify sepsis

monocytes

these type of white blood cells are Scavenger cells. They dispose of cellular debris. Their numbers increase in the late stage of acute infections and during chronic infections. Levels also rise in response to viral, bacterial, and parasitic infections. They are considered important in activating the lymphocytes and are found in the reticuloendothelial system.

Neutrophils

these type of white blood cells are phagocytes that ingest and break down foreign particles and act as an important link in generating fever.

coarse

these types of crackles can not be cleared by a patient coughing

Monocytes

these types of white blood cells are scavenger cells that dispose of cellular debris.

basophils

these types of white blood cells contain heparin and histamine in their granules, which may be important in preventing blood clotting during an inflammatory response.

eosinophils

these types of white blood cells induce Allergic reaction. They increase in response to allergic and parasitic conditions when an antigen-antibody response occurs.

Debriefs

these typically are held after an event, for example an emergency code. Common issues discussed during are what went well, what didn't go well, and what should be done differently next time

registered nurses (RN)

they assess abilities and deficits in self-care, coordinates and supports rehab through individualized plan of care and patient teachings

physical therapist (PT)

they assess mobility strengthens muscle groups and work to improve motor functions

occupational therapist (OT)

they assess the ability to preform ADL's help patients relearn basic care skills and energy conservation methods.

social worker

they coordinates placement for patients unable to remain in the home

rehabilitations physician

they direct rehabilitation and medical management of patient enrolled in the rehabilitation process

heat production

this can occur with the stimulation of the sympathetic nervous system (fight or flight)

s3

this heart sound is not normal in older adults

s2

this heart sound is the dub

s1

this heart sound is the lub

s4

this heart sound occurs because of a stiffen left ventricles

s3

this heart sounds can be heard early in diastole while ventricles are full.

atelectasis

this is a collapse or closure of a lung resulting in reduced or absent gas exchange.

Bacteremia

this is a condition of bacteria in the blood.

Ankylosis

this is a consolidation and immobilization of a joint.

isotonic exercise

this is a dynamic form of exercise with constant muscle tension, muscle contraction and active movement (maintaining equal muscle tone)

ambulatory hypertension

this is a hypertensive state while rising

superinfection

this is a secondary infection that occurs when antibiotics, immunosuppression, or cancer treatment destroy normal flora

logrolling

this is a technique used for turning patients who have had surgery or injury involving the back or the spine

venous insufficiency

this is able to reach the lower extremities but the valves are not letting the blood flow back up and is hindering blood return. ( think flakey skin, arrhythmic (red) or brown)

diastole

this is after the contraction of the heart muscles, when the chambers are filling back up with blood

ulcer

this is an open, crater-like area on the skin

tetraplegia

this is another name for quadriplegia where the patient has paralysis of the arms and legs and all muscles movement below the level of the injury

3+

this is considered to be a normal quality for a patients pulse

60-100

this is considered to be a normal rage for a patients pulse

12-20

this is considered to be the normal rate when counting a patients respiration?

36.5-37.5

this is considered to be the normal temperature in C

97.6-99.6

this is considered to be the normal temperature in F

paraplegia

this is decreased motor and sensory function to the legs

hypotonicity (flaccidity)

this is decreased muscle tone

osteoarthritis

this is degeneration of the articular surface of weight baring joints

ataxis

this is impaired muscle coordination

Replace common healthcare items with latex-free equipment

to me, I might have trouble breathing." What is the appropriate nursing action? -Assure client that balloons do not cause breathing difficulties. -Keep balloons on opposite side of the client's room. -Remind client that oranges and spinach can cause a cross-reaction. -Replace common healthcare items with latex-free equipment.

medical files, test results, and assessment data

what are secondary sources of information when collecting information about a patient?

age, morning, environment

what are some things that can cause your body temperature to be decreased

stress, hormones, evenings, smoking environment

what are some things that can cause your body temperature to be increased

increase oxygen, tidal volume, more circulation

what are the effects of exercise on the respiratory system

glasgow coma scale

what are you testing when you are making sure the patient is 3x oriented to person place and time?

men, meds (narcoitcs)

what can affect a patient's respirations to decrease

age, pain, stress, fever, shock, exercise, meds (such as adrenaline)

what can affect a patient's respirations to increase

pupils equal round reactive to light and accommodation

what does PERRLA stand for

what causes the symptom? what makes it better or worse

what does the P stand for in PQRST

how does the symptom feel, look or sound? how much of it are you experiencing now?

what does the Q stand for in the PQRST

Where does the symptom occur

what does the R stand for in the PQRST

It is the technique of listening to body sounds with a stethoscope placed on the body surface to amplify sounds

which statement describes the physical exam technique of auscultation? -It is a technique in which one or both hands are used to strike the body surface in a precise manner to produce a sound. -It is the technique of listening to body sounds with a stethoscope placed on the body surface to amplify sounds. -It is a visual examination of the client that is done in a methodical and deliberate manner. -It is the specialized use of touch for data collection.


Ensembles d'études connexes

Internet & WWW How to program - ch01-06

View Set

II Lecture Chapter 19 Short Answer: Hand Procedures pp 418

View Set

A&P 2 Ch. 20 Cardiovascular System: Vessels & Circulation

View Set

Data Structures - Mastery Quiz 4

View Set

Anatomy Chapter 12 - immunology (like mine)

View Set

psy 230 statistics of behavioral sciences exam 1

View Set